Download as docx, pdf, or txt
Download as docx, pdf, or txt
You are on page 1of 82

PSYCHOMETRICIAN SET B

GENERAL INSTRUCTIONS:

1. This test booklet contains 100 test questions.


2. Read INSTRUCTIONS TO EXAMINEES printed on your answer sheet.
3. Shade only one (1) box for each question on your answer sheets.
4. Two or more boxes shaded will invalidate your answer.
5. AVOID ERASURES.

INSTRUCTIONS:

1. Detach one (1) answer sheet from the bottom of your Examinee ID/Answer Sheet Set.
2. Write the subject title ABNORMAL PSYCHOLOGY on the box provided.

Shade Set Box A on your answer sheet if your test booklet is Set A; Set B if your test booklet is Set B.

1. What manifestations is usually the first indication of the onset of AD?

A. Inability to perform ADLs

B. Sundowning at night

C. Subtle memory deficits

D. Inability to communicate

Rationale: C. Memory deficits are usually the first indication of AD, are subtle and may not be noticed by
friends and family until the client exhibits unsafe behaviors. The other answers listed are symptoms in
later stages of AD.

2. The psychologist also notes that Lolo Mel has also begun to experience speech deficits and
incontinence. How would the psychologist interpret these findings?

A. The client is in stage I

B. The client is in stage II

C. The client is in stage III

D. The client is in stage IV

Rationale: B. Stage II or the middle stage of AD, is characterized by complete disorientation to time,
place, and events. The client develops speech and language difficulties, as well as incontinence.

3. A patient with Alzheimers disease (AD) in a long-term care facility is wandering the halls, very
agitated, asking about her family, and crying. The best action by the psychologist is to
PSYCHOMETRICIAN SET B

A. ask the patient, why are you behaving this way?

B. tell the patient, lets go get a snack in the kitchen.

C. ask the patient, wouldnt you like to lie down now?

D. tell the patient, just take some deep breaths and calm down.

Rationale: B. Patients with late Alzheimers disease (AD) frequently become agitated, but because there
short-term memory is so pronounced, distraction is a very good way to calm them. Why questions are
upsetting to them because they dont know the answer, and they cannot respond to normal relaxation
techniques.

Situation: Ali, a 59 year-old retired boxer, has been diagnosed with Parkinsons disease and is admitted
to the Psychological unit.

4. When evaluating Ali, a psychologist observes for which condition?

A. Bulging eyeballs

B. Diminished distal sensation

C. Increased dopamine levels

D. Muscle rigidity

Rationale: D. Parkinsons disease is characterized by the slowing of voluntary muscle movement,


muscular rigidity, and resting tremor.

5. Which statement best describes the cause of Parkinsons disease?

A. Loss of the myelin sheath surrounding peripheral nerves

B. Degeneration of the substantia nigra, depleting dopamine

C. Bleeding into the brain stem, resulting in motor dysfunction

D. An autoimmune disorder that destroys acetylcholine receptors

Rationale: B. Parkinsons disease is caused by degeneration of the substantia nigra in the basal ganglia of
the brain, where dopamine is produced and stored.

6. Which symptom occurs initially in Parkinsons disease?

A. Akinesia

B. Aspiration of food

C. Dementia

D. Pill rolling movements of the hand


PSYCHOMETRICIAN SET B

Rationale: D. Early symptoms of Parkinsons disease include coarse resting tremors of the fingers and
thumb. Akinesia and aspiration are late signs of Parkinsons disease. Dementia occurs in only 20% of the
clients with Parkinsons disease.

7. Which of the following neurotransmitters is decreased in patients with Parkinsons disease?

A. Acetylcholine

B. Norepinephrine

C. Serotonin

D. Dopamine

Answer D. Dopamine is the neurotransmitter that is decreased in patients with Parkinsons disease.

8. Which is a common cognitive problem associated with Parkinsons disease?

A. Emotional liability

B. Depression

C. Memory deficits

D. Paranoia

Rationale: C. Memory deficits are cognitive impairments. The client may also develop a dementia.

Anxiety is one of the bodys responses to maintain homeostasis. However, when anxiety cannot be dealt
with appropriately, it becomes a maladaptive response.

9. A client with moderate anxiety is most likely to exhibit which of the following signs and
symptoms?

A. Hypervigilance and increased blood pressure

B. Increased heart rate and decreased ability to concentrate

C. Increased heart rate and decreased blood pressure

D. Sweaty palms and increase salivation

Rationale: B. Moderate anxiety is typically accompanied by an increased heart rate along with signs of
diminished ability to concentrate (selective inattention, difficulty remaining attentive, decreased
learning ability). Hypervigilance, a state in which an individual scans his environment for threats, is more
common in severe anxiety or panic episodes than in moderate anxiety. Besides increased heart rate,
other physiologic manifestations of anxiety include increased blood pressure, sweaty palms, and dry
mouth. Increased salivation is not associated with this disorder.

10. When intervening with a client who has severe or panic-level anxiety, it is essential for the
psychologist to:
PSYCHOMETRICIAN SET B

A. become aware of and control her own feelings of anxiety.

B. have the client sit down and be quiet, and give detailed instructions to him.

C. call security to restrain the client.

D. leave the client alone so he well become quiet.

Answer A.

11. A client reports a vague feeling of apprehension and states, I feel scared. He also has an increased
pulse rate and blood pressure. Which of the following diagnoses is most appropriate in this situation?

A. Anxiety

B. Fear

C. Ineffective coping

D. Disturbed sleep pattern

Answer A.

12. When caring for a client with panic level anxiety, the psychologist should consider which expected
outcome to be a priority?

A. The client will discuss his concerns for 10 minutes, three times a day.

B. The client will verbalize a decreased level of anxiety by the end of the hospitalization.

C. The client will verbalize a decreased level of anxiety within 2 hours.

D. The client will state the name and dose of the anxiolytic medication.

Answer C.

Situation: The family members of a patient just diagnosed with personality disorder receives an
orientation from the psychiatrist about the nature of the said disorder.

13. The client exhibiting mistrust, guardedness, and restricted affect is showing signs of which
personality?

A. Antisocial personality disorder

B. Dependent personality disorder

C. Narcissistic personality disorder

D. Paranoid personality disorder

Answer D.
PSYCHOMETRICIAN SET B

14. The axis II diagnosis of a client is schizoid personality disorder. Which approach should the
psychologist plan to use when interacting with this client?

A. Helpful and nurturing

B. Matter-of-fact and calm approach

C. Light and playful

D. Warm and friendly

Answer B.

15. A client diagnosed with paranoid personality disorder needs information regarding medications.
Which psychological intervention would assist this client in understanding prescribed medications?

A. Ask the client to join the medication education group

B. Provide one-on-one teaching in the clients room.

C. During rounds, have the physician ask if the client has any questions.

D. Let the client read the medication information handout.

Answer B

16. When caring for a client with schizoid personality disorder, the psychologist primarily focuses on
which of the following?

A. expanding the clients interest in objects and things

B. increasing the clients ability to experience pleasure

C. increasing solitary activities

D. improving the clients functional relationships

Answer D.

17. Which defense mechanism is being used by a paranoid client who blames others for problems he is
experiencing?

A. projection

B. displacement

C. rationalization

D. intellectualization

Answer A.
PSYCHOMETRICIAN SET B

18. When working with a client with paranoid personality disorder, the psychologist will find that the
client most often resorts to which one of the following defense mechanisms?

A. denial

B. projection

C. sublimation

D. repression

19. When assessing the client with a diagnosis of schizoid personality disorder, you will find a pervasive
pattern of detachment from social relationships and, additionally, which of the following?

A. extreme anger when sufficiently provoked by others

B. a restricted range of emotional expression with others

C. seeking approval from no more than two other persons

D. more than the average interest in sexual activity with strangers

Answer B.

20. You are assigned to a client with schizotypal personality disorder. Your assessment would likely
reveal which of the following behaviors?

A. seeks attention and engages in erratic behavior

B. withdrawn and engages in odd, eccentric behavior

C. overtly psychotic and experiencing hallucinations and delusions

D. active participation in activities with other clients and staff

Answer B.

21. A client has a diagnosis of schizoid personality disorder. When assessing client, the psychologist
should expect that the clients behavior would be:

A. Rigid and controlling

B. Dependent and submissive

C. Detached and socially distant

D. Superstitious and socially anxious

Answer C.

Situation: The Psychologist has just been informed that a new client on the unit has a diagnosis of
schizophrenia, paranoid type.
PSYCHOMETRICIAN SET B

21. Based on this limited information, the psychologist knows that she will need to address which basic
need first?

A. Esteem and recognition

B. Love and belonging

C. Physiologic integrity

D. Safety

Answer D.

22. When assessing a client with paranoid schizophrenia, which findings should be the psychologists
immediate concern?

A. Immobility, clanging, altered sleep patterns, and difficult family relationships.

B. Problems with role-functioning, peculiar mannerisms, and decreased social competence.

C. Stressful life events, altered sleep patterns, fatigue, and hostility.

D. Paranoid delusions, increased suspiciousness, and hostility.

Answer D.

23. When interacting with a client with a schizophrenic disorder, the psychologist should remember to:

A. avoid interactions whenever possible.

B. have an intensive 1-hour counseling sessions twice a day.

C. speak loudly and clearly.

D. use brief, simple statements.

Answer D.

24. An 18-year-old man is admitted to the inpatient unit with a diagnosis of schizophrenia. As the
psychologist approaches him, she notices that he is grimacing and talking to the wall. The psychologists
first action should be to:

A. ask him if he is hearing voices.

B. ask him to walk in the hall.

C. engage him in conversation.

D. have him placed in seclusion

Answer A.
PSYCHOMETRICIAN SET B

25. The client states, I am low, how low can you go, Im going to get married. Yeah, Im going to marry
Bill. Ive got lots of bills to pay after that last spree! Which observation is the psychologist likely to
document about this client?

A. Circumstantiality was noted.

B. Disorientation was noted.

C. Flight of ideas was noted.

D. Speech patterns using word salad was noted

Answer C.

Situation: A newly admitted patient with schizophrenia tells you that her dead grandmother sits in a
chair in her room and tells her when it is safe to leave the room.

26. Using therapeutic communication, what is the psychologists best response to the client?

A. If your grandmother is dead, she cannot be in your room

B. Your grandmother must have been very special to you

C. You dont feel safe here?

D. We are here to help you. You dont need your grandmother for protection.

Answer C

27. Which of the following illustrates a common symptom of schizophrenia?

A. During a discussion of the loss of significant people in her life, the client is laughing
uncontrollably.

B. The admission history describes the clients affect as appropriate and the mood as depressed.

C. The client has strong feelings about a decision to go to supervised housing.

D. The client is an active participant in verbal groups on the unit.

Answer A.

28. A client with schizophrenia states, The TV screen is constantly communicating with me. Which of
the following would the psychologist document?

A. Delusions of grandeur

B. Ideas of influence

C. Ideas of reference

D. Looseness of association
PSYCHOMETRICIAN SET B

Answer C.

29. The belief expressed by the client that an alien is creating sores on his body with a laser is classified
as a (n):

A. Hallucination

B. Neologism

C. Ideas of reference

D. Delusion

Answer D.

30. If a client were experiencing negative symptoms of schizophrenia, the GP would expect to see:

A. Flat affect and little speech

B. Rigid posture

C. Excessive purposeless movements

D. Inappropriate laughter

Answer A.

Situation: Micalyn, a new psychologist assigned in the psychiatric unit is ever conscious of the ethico-
moral concerns that affect the performance of her duties in the said unit.

31. The principle of beneficence in ethical decision making is best reflected in which of the following
statements?

A. Be honest and open and tell the truth at all costs.

B. Be faithful to promises and obligations.

C. Treat people fairly and distribute resources equitably.

D. Promote good, do no harm, and prevent harm.

Answer D.

32. A psychologist is explaining the Bill of Rights of psychiatric patients to a client who has voluntarily
sought admission to an inpatient psychiatric facility. The following rights should be included in the
discussion except:

A. right to select health care team and psychiatric members

B. right to refuse treatment

C. right to a written treatment plan


PSYCHOMETRICIAN SET B

D. right to confidentiality

Answer A.

33. Joe is very restless and is pacing a lot. The psychologist says to Joe, If you dont sit down in the chair
and be still, Im going to put you in restraints! The GP may be charge of:

A. assault

B. battery

C. defamation of character

D. false imprisonment

Answer A.

Situation: Gary, a 35-year old, has been an alcoholic for 5 years. He recently lost his job due to his
frequent tardiness and has spent up all his savings. Despite this, he doesnt admit that he has a problem,
so his wife brought him to the Rehabilitation Center.

34. Gary states, I dont think my drinking has anything to do with why I am here in the hospital. I think I
have problems with depression. Which statement by the psychologist is the most therapeutic
response?

A. I think you really need to look at the amount you are drinking and consider the effect on your
family.

B. Thats wrong. I disagree with that. Your admission is because of your alcohol abuse and not for
any other reason.

C Im sure you dont mean that. You have realized that alcohol is the root of you problems.

D. I find it hard to believe that alcohol is not a problem because you have recently lost your job
and your drivers license.

Answer D.

35. During a GP-client interaction, Gary shares with you that he hits his wife while intoxicated with
alcohol. He asks the psychologist, Mapapatawad pa ba niya ako?: Which of the following responses by
the GP would be best in this situation?

A. Malalaman mo lang ang sagot sa tanong mo kung lalapitan mo siya.

B. Maaari nating pag-usapan iyan sa ating family session.

C. Depende sa kaniya kung mahal ka niya talaga.

D. Sa tingin ko may dinaramdam ka dahil nasaktan mo ang asawa mo.

Answer D.
PSYCHOMETRICIAN SET B

36. A client diagnosed with alcoholism is admitted to substance abuse unit complaining of decreased
exercise tolerance, lower extremity edema, arrhythmias, and dyspnea. Which intervention as a
psychologist therapist would be appropriate for this client?

A. Providing thiamine-rich foods

B. Administering digoxin (Lanoxin) and furosemide (Lasix)

C. Reorienting the client to person, place, and time.

D. Encouraging high-sodium foods.

Answer B.

37. A Psychologist has developed a relationship with a client who has an addiction problem. The
following actions would indicate that the therapeutic intervention is in the working phase except:

A. the client discusses how the addiction has contributed to family distress

B. the client verbalizes difficulty identifying personal strengths

C. the client discusses the financial problems related to the addiction

D. the client expresses uncertainty about what topic to discuss

Answer D.

Situation: Everyday the psychologist is faced with situations related to death and dying and grief and
grieving. As a registered psychologist deals with the varied reactions of patients and relatives to the
phenomenon of loss, the GP must also be in touch with his own concept of mortality.

38. Maria has been in a comatose state for the past 8 months as a result of an automobile accident.
Although doctors have told her husband, Reuben, that there is no brain function, Reuben insists that she
is showing responses. Which of the following stages of grief is Reuben experiencing?

A. Bargaining

B. Anger

C. Denial

D. Depression

Answer C.

39. A client recently lost his spouse. Which behavior indicates that the client is going through normal
stage of grieving?

A. The client starts using chemicals

B. The client becomes an overachiever


PSYCHOMETRICIAN SET B

C. The client shows signs of hyperactivity

D. The client shows a loss of warmth when interacting with others

Answer D.

40. A female client, who has been told by her physician that she has untreatable metastatic carcinoma,
tells the psychologist that she believes the physician has made an error, she does not have cancer, and
she is not going to die. The psychologist evaluates that the client is experiencing the stage of death and
dying known as:

A. Anger

B. Shock

C. Bargaining

D. Acceptance

Answer B.

41. Immediately after the death of their 3-year-old son the psychologist would be most therapeutic by
asking the parents:

A. Do you feel ready to consent to an autopsy?

B. Have you made a decision about organ donation?

C. Would you like to talk about how you will tell your other children?

D. Can I be of some help with any practices that are important to you?

Answer D.

42. Shortly after the death of her husband following a long illness, the wife visits the mental health
clinic complaining of malaise, lethargy and insomnia. The psychologist, knowing that it is most important
to help the wife cope with her husbands death, should attempt to determine the:

A. Age of the wife

B. Timing of the husbands death

C. Socioeconomic status of the couple

D. Adequacy of the wifes support system

Answer D.

Situation: Human being use various defense mechanism to ensure the integrity of the ego.

43. A person trying to lose weight decides to take a 15 minute walk every time the temptation to snack
between meals occurs. This demonstrates which of the following defense mechanisms?
PSYCHOMETRICIAN SET B

A. Displacement

B. Projection

C. Rationalization

D. Sublimation

Answer D.

44. Ariel, a student, went to watch a movie with a friend the night prior to the unit exam. The next
morning he received a low score on the exam. After seeing the score, Ariel slammed the book on the
table and says to his classmate, "Our research is taking up much of our time and I don't have time to
study! Anyway, I would not have mattered anyway because my teacher is unreasonable." Which of the
following defense mechanisms is Ariel using?

A. compensation, rationalization, denial

B. reaction formation, conversation, projection

C. rationalization, displacement, projection

D. displacement, regression, intellectualization

Answer C

45. The relative confirmed that ever since his father died, he began talking, dressing up, and acting like
his father. This is:

A. Introjection

B. Sublimation

C. Reaction Formation

D. Projection

Answer A.

46. Your client, prior to admission at the Remotivation Ward disclosed that he was troubled by
homosexual urges and initiated a campaign in the Senate campaigning not to grant homosexuals rights
to petition and assembly. Based on your previous lectures, you learned that this is:

A. Sublimation

B. Reaction Formation

C. Denial

D. Undoing

Answer B.
PSYCHOMETRICIAN SET B

Situation: Melissa, a new GP, admitted several suicidal patients in the last 3 days.

47. A client expressing suicidal thoughts suddenly becomes very calm and has a much improved outlook
on life. The GP recognizes that this behavior typically indicates:

A. improved coping skills.

B. increased self-esteem.

C. an increased suicide risk.

D. a positive response to treatment.

Answer C.

48. A client was admitted to the psychiatric unit after a suicide attempt. Which of the following
statements by the client would lead the psychologist to suspect that another suicide attempt may be
imminent?

A. How often does the staff make rounds.

B. I dont want to be alone right now.

C. There is something to do here.

D. When will I be discharged?

Answer A.

49. When children talk about wanting to kill themselves, parents need most to:

A. get help for the child and take the idea of suicide seriously

B. reduce the amount of television that the child views daily

C. send the child to her room for a time out and to think

D. ignore this kind of attention-seeking behaviour in their child

Answer A.

50. A male client is brought to the psychiatric emergency department after attempting to jump off a
bridge. The clients wife states that he lost his job several months ago and has been unable to find
another job. The primary intervention as a psychologist at this time should be to assess for:

A. Feelings of failure

B. A history of depression

C. Plans of committing suicide

D. The presence of marital difficulties


PSYCHOMETRICIAN SET B

Answer C.

Situation: Matet, a 44 year old former government employee, had been in and out of the hospital due to
his Bipolar Disorder. He is pacing constantly today while other clients are having a birthday party. There
is music, noise and food. The client walks over to the table and starts grabbing handfuls of cake to eat as
he paces up and down the halls.

51. Among the following, what is the best response of the GP to this behavior?

A. Let him continue to pace and eat

B. Medicate him with anxiety drug

C. Restrain him in his room

D. Invite him to go outside and take a walk with the psychologist

Answer D.

52. Clients experiencing a manic episode will most likely be noncompliant with treatment because:

A. They do not realize they need treatment

B. They are too busy

C. They want to be liked by others

D. They enjoy the high

Answer D.

53. The GP learns a client is a college sophomore who recently transferred to the local university from
another school in his home state. He reports he has been feeling depressed since he arrived and tells the
GP, I cant seem to get with it... I dont know anyone here and cant get interested in my classes.
Which of the following is an appropriate diagnosis?

A. guilt related to failure to achieve his goals

B. inability to cope related to loss or separation from loved ones

C. feelings of hopelessness related to change of residence

D. low self-esteem related to lack of trust

Answer B.

54. A male college student, age 19, is admitted to the psychiatric unit with complains of suicidal
thoughts and plan to hang himself. Which of the following actions is not a priority for the GP?

A. remove his clothing, have him put on a hospital gown, and admit him to a seclusion room until a
complete assessment of suicide risk can be done
PSYCHOMETRICIAN SET B

B. contact a family member to ask if he has made any prior suicide attempts

C. place the client on suicidal precautions

D. encourage the client to discuss recent events which led him to feel so hopeless

Answer A.

55. Before she is discharged from the hospital, the client and her husband attend a client education class
on the topic of depression. They learn about the behaviors that could indicate a recurrence of
depression. These could include all of the following except:

A. psychomotor retardation

B. grandiosity

C. self-devaluation

D. insomnia

Answer D.

56. The psychiatrist orders tranylcypromine (Parnate) for a depressed client who has not responded to
tricyclics. The psychologist knows that dietary teaching is essential. She should instruct the client to
avoid all of the following substances except:

A. beer and red wine

B. cheddar cheese and sausage

C. cottage cheese and canned peaches

D. liver and Italian green beans

Answer C.

57. A psychologist instructor is teaching about the cause of mood disorders. Which statement by a
psychology student best indicates an understanding of the etiology of mood disorders?

A. When clients experience loss, they learn that it is inevitable and become hopeless and
helpless.

B. There are alterations in the neurochemicals, such as serotonin, which cause the clients
symptoms.

C. Evidence continues to support multiple causations related to an individuals susceptibility to


mood symptoms.

D There is a genetic component affecting the development of mood disorder.

Answer C.
PSYCHOMETRICIAN SET B

58. In the space of 5 minutes, the client has been laughing and euphoric, then angry, then crying for no
reason that is apparent to the GP. This behavior would be best described as:

A. Flight of ideas

B. Lack of insight

C. Labile mood

D. Tangential thinking

Answer C.

59. A client with recurrent headaches has been told by the physician that the cause is likely
psychosomatic. The client reports this conversation to the GP and says, That just cant be true! My
head hurts so bad sometimes that it makes me sick to my stomach. The GPs best response is:

A. To give the client some privacy and time to calm down.

B. To say nothing and sit quietly with the client.

C. The pain in your head is very real.

D. Well, thats not what your doctor thinks.

Answer C.

60. During assessment, the psychologist will find that the client who has a diagnosis of body dysmorphic
disorder will have which of the following symptoms?

A. Dissatisfaction with body shape and size

B. Preoccupation with an imagined defect in appearance

C. Fantasizing oneself as changed into an ideal appearance

D. A history of eight or more plastic surgeries on various body parts

Answer B.

61. The primary gain from a conversion symptom is best represented by which of the following?

A. Feelings of being important to family, caregivers, and others

B. Attention from the caregivers at home, friends, or health care staff

C. Getting out of negative, undesired, or stress-producing responsibilities

D. Reduction of anxiety by resolution of an unconscious psychological conflict

Answer D.
PSYCHOMETRICIAN SET B

62. When scenario best typifies a client with somatoform disorder?

A. A client describes multiple gastrointestinal complaints without organic pathology.

B. A client exhibits a morbid preoccupation with the fear of cancer.

C. A client has recurrent episodes of intense anxiety that keep her from activities.

D. A client recognizes an overwhelming and uncontrollable fear of episodes.

Answer A.

63. La belle indifference is a common manifestation of which disorder?

A. Conversion disorder

B. Dissociative disorder

C. Posttraumatic stress syndrome

D. Somatization disorder

Answer A.

64. Client Myrna is admitted to a mental health unit with a diagnosis of dissociative fugue. Myrna left
home 2 days earlier and was found in another city yesterday, and she has no memory of traveling to the
other city. After the initial interview, the clients spouse asks the GP about the loss of memory. Which
statement made by the GP is appropriate?

A. Your spouse must be unhappy in the marriage is seeking a way out.

B. The loss of memory regarding the travel experience is common with this disorder.

C. The fugue state occurred because your spouse has developed an alternate personality.

D. Im sure your spouse remembers the traveling but just doesnt want to talk about it right now.

Answer B.

65. When assessing a client with dissociative identity disorder, the GP is most likely to find the client
describing which of the following things?

A. Feeling everything is speeded up, as if in a manic state

B. A total loss of control over life and relationships with others

C. Finding purchases or belongings for which they cannot account

D. Being able to identify and call on all the alters, at will, for protection

Answer C.
PSYCHOMETRICIAN SET B

66. The person with dissociative amnesia will have the inability to recall which of the following things?

A. Events in the immediate past

B. Significant personal information

C. Information about significant others

D. Skills such as balancing a checkbook

Answer B.

67. A client is mildly agitated walking to the exit of the hospital unit. When attempting to redirect an
agitated and confused client, the GP should take which approach?

A. Approach the client calmly, explaining that you will touch him.

B. Call the client from a distance in a loud voice.

C. Change your activity so the client will be distracted.

D. Quickly approach the client and take the client by the arm.

Answer A.

68. Which of the following is possible from psychological factors, such as anxiety?

A. can contribute to both the development and progress of heart disease

B. can cause heart disease

C. cannot cause heart disease but does affect the clients prognosis

D. can affect the clients ability to adjust to a diagnosis of heart disease but has little direct effect
on treatment outcomes

Answer A.

69. The GP can evaluate the agoraphobic clients progress as improving when the client is able to attend
which of the following activities?

A. daily community/milieu meetings

B. occupational therapy on the unit

C. the hospital gift shop

D. a unit picnic in a local park

Answer D.

70. Which of the following is hyperventilation frequently a sign of?


PSYCHOMETRICIAN SET B

A. fatigue

B. opiate withdrawal

C. anxiety

D. petit mal epilepsy

Answer C.

71. A psychologist is assessing a client diagnosed with dependent personality disorder. Which
characteristic is a major component of this disorder?

A. Abrasive to others

B. Indifferent to others

C. Manipulative of others

D. Overreliance on others

Answer D.

72. A client with obsessive-compulsive personality disorder will probably exhibit:

A. an exaggerated sense of self-importance

B. a lack of concern about right and wrong

C. rigidity and a moralistic attitude

D. rigidity, temper tantrums, and impulsiveness

Answer C.

73. You are observing a client who has to check the door to see if it is locked at least 50 times before
leaving the house. The client you are observing is experiencing which of the following?

A. obsessions

B. compulsions

C. preservation

D. preoccupations

Answer B.

74. A client diagnosed with obsessive-compulsive personality disorder is admitted to a psychiatric unit in
a highly agitated state. The physician prescribes a benzodiazepine. Which medication is classified as a
benzodiazepine?
PSYCHOMETRICIAN SET B

A. Clonazepam (Klonopin)

B. Lithium carbonate (Lithium)

C. Clozapine (Clozaril)

D. Olanzapine (Zyprexa)

Answer A.

75. Which of the following is an appropriate intervention for controlling the behavior of the suicide
personality, Beth.

A. When Beth emerges, put the client in restraints

B. Keep Ellen in isolation during her hospitalization

C. Make a verbal contract with Ellen that Beth will do no harm

D. Elicit the help of another, strong-willed personality to help control Beths behavior

Answer D.

76. An obese client continues to gain weight. The psychologist observes the client obtaining candy and
snacks from other clients and plans to use confrontation in her interaction with the client. It will be
important for the GP to:

A. ask the client why he is behaving in this manner.

B. focus on the clients explanation of his behavior.

C. inform the client that his 24-hour calorie totals will be reduced.

D. point out the behavior as close to the snacking as possible.

Answer D.

77. The GP realizes that typical characteristics of girl with anorexia nervosa are which of the following?

A. they fail to comply with their parents wishes or societal expectations

B. they exercise relentlessly

C. they are truthful in reporting their eating habits

D. they have problems with self-control

Answer B.

78. A 16-year-old, 54 white high school student is admitted to an eating disorders program by her
psychiatrist. She tells the psychologist she has lost 25 pounds over the past month and now weight 85
PSYCHOMETRICIAN SET B

pounds. The psychologist assessment identifies several behaviors that are characteristic of clients in the
beginning stages of anorexia nervosa. What are characteristics of clients with anorexia nervosa?

A. appetite loss, amenorrhea, bradycardia, loss of 15% of pre illness body weight

B. appetite loss, amenorrhea, tachycardia, hyperactivity

C. tachycardia, insomnia, fear of obesity, bulimia

D. amenorrhea, bradycardia, disturbed body image, loss of 15% of pre-illness body weight

Answer D.

79. The alcoholic client tells the psychologist he has not had anything to drink for 24 hours prior to
admission. He complains of feeling anxious and shaky. Based on her knowledge of alcohol withdrawal,
what other behaviors could the GP expect him to display during the early phase of alcohol withdrawal?

A. coarse tremors, tachycardia, insomnia

B. confusion, visual hallucinations, delusions

C. disorientation, confabulation, memory deficits

D. incoordination, impaired thinking, irregular eye movements

Answer A.

80. Cocaine intoxication can cause all of the following except:

A. seizures

B. hyperactivity

C. paranoid behavior

D. nystagmus

Answer D.

81. Ellen suddenly has changed in voice quality and sentence structure. What is the most therapeutic
response by the GP?

A. You must be feeling very needy

B. I wonder why youre not acting your age

C. Can you tell me what is happening

D. This behavior keeps you from working on your problem

Answer C.
PSYCHOMETRICIAN SET B

82. Carol, an adolescent, just returned from group therapy and is crying. She says to the GP, All the
other kids laughed at me! I try to fit in, but I always seem to say the wrong thing. Ive never had a close
friend. I guess I never will. Which is the most appropriate response by the GP?

A. Why dont you feel this way about yourself?

B. What makes you feel this way about yourself?

C. The next time they laugh at you, you should just get up and leave the room!

D. Keep your chin up and hand in there. Your time will come.

Answer B.

83. A client who has just been diagnosed with cancer tells the GP that her would rather be dead than go
through the treatment for cancer. The most appropriate response is:

A. What it is about the cancer treatment that concerns you?

B. If you dont receive the treatment, you will get your wish.

C. Why dont you talk to your doctor about your feelings?

D. That wouldnt be fair to your family, would it?

Answer A.

84. The GP enters a clients room. The clients son tells the GP, You people cant do anything right. Ever
since my father was admitted to this hospital, it has been one mistake after another. I am taking him out
of here before you kill him. The most therapeutic response by the GP is which of the following?

A. You feel that your father is not being well taken care of.

B. We have the best intentions for the client.

C. Ill get the supervisor for you.

D. Your father hasnt complained about the care. What specifically is the problem.

Answer A.

85. A client has been told by the doctor that he has cancer and that it has advanced so far that
treatment will not help. When the psychologist enters the room to set up his bath equipment, he says to
the psychologist, Im not an invalid, you know. I can take care of myself. Get out and leave me alone.
Which of the following is the best response?

A. I know that you are not an invalid. However, I was trying to help you.

B. It sounds to me like you are angry about something. Did somebody do something wrong?

C. You are pretty upset. Lets talk about it.


PSYCHOMETRICIAN SET B

D. Ill just set up this equipment for you to bathe youre not so angry.

Answer C.

86. A psychiatric mental health professional who predominantly uses the intervention mode of group and
family therapies has the goal of:
A. changing the dynamics of the clients behavior
B. improving the quality of the individuals interpersonal interactions
C. developing insight
D. managing the symptoms

Answer B

87. The psychologic response that best addresses his needs when he tells the GP about his delusion would
be:
A. How is it that you are so important that terrorists want to kill you?
B. Your thinking is distorted because your brain chemicals are out of balance.
C. Your story is so fantastic that I cannot believe its true.
D. It must be frightening to think you are being targeted.

Answer D

88. The GP would evaluate that a patient who has auditory hallucinations has improved when the patient
can:
A. tell the psychologist what the voices say. C. validate what is real.
B. tell the voices to be quiet. D. do what the voices command.

Answer C

89. A young patient with schizophrenia is standing in his pajamas next to the shower. The GP observes
that he seems dazed and indecisive. The most helpful intervention would be to say:
A. Its time for you to take your shower.
B. Take off your pajamas and step into the shower.
C. Is something wrong?
D. Why are you waiting to get into the shower?

Answer B

90. A newly admitted patient has the diagnosis of catatonic schizophrenia. The psychologist would expect
to assess:
A. psychomotor symptoms. C. inappropriate affect
B. intense suspiciousness. D. clanging communication.

Answer A

91. The GPs reply should be predicated on the knowledge that dissociative identity disorder is thought to
be related to:
PSYCHOMETRICIAN SET B

A. faulty learning. C. genetic predisposition.


B. severe childhood trauma. D. intentional production of symptoms

Answer B

92. The GPs who is addressing memory problems with a patient with a dissociative disorder can be most
effective if he or she:
A. reorients the patient to time, place, and person at every contact.
B. observes for cues that the patient is ready to receive information.
C. instructs the patient not to be overly concerned with memory loss because no organic
pathology exists.
D. tells the patient of the events surrounding the memory loss at the initial therapy session.

Answer B

93. When working with a patient who has dissociative amnesia, the psychologist should plan to begin by:
A. taking measures to prevent identity diffusion.
B. setting mutual goals for behavioral changes.
C. helping the patient develop a realistic self-concept.
D. identifying and supporting patient strengths

Answer C

94. The symptom the psychologist can expect a patient with dissociative fugue to manifest is:
A. the notion that some part of the body is ugly or disproportionate.
B. a feeling of detachment from ones body.
C. worry about having a serious disease.
D. travel away from home and assumption of a new identify.

Answer D

95. A client says, I dont think Id like to go anywhere with any member of my family. They make me sick.
The GPs most facilitative response is:
A. I know what you mean; sometimes I feel the same way.
B. Your family makes you sick? Why is that?
C. What do you mean by your family making you sick?
D. It seems that you are really sick of your family. What do you want to do about it?
Answer C

96 . A client says, Im really angry. How should the psychologist respond therapeutically?
A. What are you angry about? C. Why are you angry?
B. Dont worry. Everyone gets angry sometime. D Youre really angry.

Answer A

97. One of the clients in the unit requires consent. Which one the following examples illustrate informed
consent?
PSYCHOMETRICIAN SET B

A. The doctor informs the client of the treatment, the client is competent, and then signs the
consent.
B. The client is competent, is given choices as well as the benefits and limitations for each choice,
and is not forced to make the decision.
C. The client is given the options and the opportunity to ask questions.
D. The client is competent and able to make informed choices.

Answer B

98. An individual who recently celebrated his 65th birthday is planning to leave from the job he has held
for the last 35 years. His place of employment is providing information on how to adjust to the
change in lifestyle. The industrial psychologist leading the workshop is aware that if the individual
does experience an adjustment disorder, it will likely be related to:
A. loss of identity and purpose. C. boredom from having few interests.
B. concern about finances. D. loneliness from having to spend time
alone.

Answer A

99. An action the psychologist can advise a family to take in the home setting to enhance safety for the
family member with Alzheimers disease is:
A. placing throw rugs on tile or wooden floors.
B. instructing patient on cooking safety.
C. allowing patient to smoke unattended.
D. having patient wear an identification bracelet with name, address, and telephone number.

Answer D

100. Which of the following techniques is appropriate for successful interaction with a patient who has
been diagnosed with Alzheimers disease?
A. Giving all directions at one time to increase understanding
B. Correcting errors made by the patient by speaking to him in a loud, clear voice
C. Encouraging communication and maintaining a calm demeanor
D. Setting strict time limits and repeatedly rephrasing misunderstood questions

Answer C
PSYCHOMETRICIAN SET B

GENERAL INSTRUCTIONS:

6. This test booklet contains 100 test questions.


7. Read INSTRUCTIONS TO EXAMINEES printed on your answer sheet.
8. Shade only one (1) box for each question on your answer sheets.
9. Two or more boxes shaded will invalidate your answer.
10. AVOID ERASURES.

INSTRUCTIONS:

3. Detach one (1) answer sheet from the bottom of your Examinee ID/Answer Sheet Set.
4. Write the subject title THEORIES OF PERSONALITY on the box provided.

Shade Set Box A on your answer sheet if your test booklet is Set A; Set B if your test booklet is Set B.

1. When we use the term personality in everyday speech, we are usually referring to
a) unconscious forces.
b) an unorganized pattern of the perception of I.
c) that which determines an individuals behavior.
d) an individuals public image or social role.

2. According to Freud, during what stage does the superego emerge?


(A) Genital
(B) Oral
(C) Phallic
(D) Anal
3. Attributing ones own unacceptable thoughts or feelings to someone else is called what?
(A) Reaction formation
(B) Projection
(C) Displacement
(D) Sublimation
4. What is the part of the personality that compels people to act in perfect accordance with moral
ideals?
(A) The id
(B) The superego
(C) The pleasure principle
(D) The reality principle
5. Which theorist focused on the importance of the self-concept in personality?
(A) Carl Rogers
(B) Alfred Adler
(C) Walter Mischel
(D) Hans Eysenck
PSYCHOMETRICIAN SET B

6. Which one of the following statements about humanistic theories is false?


(A) They do not provide insight into the evil side of human nature
(B) They are biased because they are based on individualistic values
(C) They are based on studies of people with psychological disorders
(D) They are difficult to test empirically
7. Studies of temperament provide evidence for which of the following?
(A) The concept of congruence
(B) Maslows concept of the self-actualizing person
(C) Sublimation
(D) Genetic contributions to personality
8. When he was young, Gregs father was consistently promoted at work for his diligence. Greg saw this
and learned to be a conscientious worker himself. This fact could most easily provide evidence for
whose theory of personality?
(A) Sigmund Freud
(B) Albert Bandura
(C) Abraham Maslow
(D) Hans Eysenck
9. What is an advantage of projective personality tests?
(A) Different people would score them the same way
(B) They are very reliable
(C) They are not transparent to the subject
(D) They have lie scales that indicate whether subjects are being deceptive
10. Which of Jungs beliefs differed from Freuds theories?
(A) Everyone has a collective unconscious
(B) The unconscious is important in shaping personality
(C) The main motivation for behavior is a striving for superiority
(D) Behavior depends on the consequences that follow it
11.Carl is obsessed with cleanliness and control. Which stage of Freuds Stages of Psychosexual
Development has Carl become fixated at?
Please choose from one of the following options.
a. Oral Stage
b. Phallic Stage
c. Latency Stage
d. Anal Stage
12.The id, ego, and superego can be best characterized as:
a. Please choose from one of the following options.
b. Part of the neocortex
c. Located in the occipital lobe
d. Systems that make up personality
13. According to Freud, which part of our personality understands that other people have needs and
that being selfish can hurt us in the future?
a. Id
b. Ego
c. Superego
d. None of the above
PSYCHOMETRICIAN SET B

14. Which of the following personality theorists considered the concept of personality to be
unnecessary?
a) Gordon Allport
b) Sigmund Freud
c) Carl Rogers
d) B. F. Skinner

15.Which approach to personality sees personality as something which is measurable?

a) Ideographic
b) Nomothetic
c) Social-radical
d) All of the above

16.Which of the following statements would be true of a person with, in Jung's (1923) terms, an
extraverted personality?

a) They gain energy from social situations


b) Social situations drain them of energy
c) They get energy from within
d) They thrive on working alone

17.Which personality measuring instrument uses four scales and is derived from the work of Jung?

a) Cattell's 16PF traits


b) Myers-Briggs Type Indicator
c) Big five personality factors
d) Keirsey's temperament sorter

18.Which of the following best describes Keirsey's (1988) temperament sorter?

a) It was the inspiration for the Myers-Briggs Type Indicator.


b) It is a direct critique of the Myers-Briggs Type Indicator.
c) It links Myers-Briggs Type Indicator results to appropriate careers.
d) It is used to analyse Myers-Briggs Type Indicator results when filling job vacancies.

19.How are the big five personality factors (McRae and Costa, 1990, 1996) also known?

a) RIVER scale
b) BROOK scale

c) PONDS scale
d) OCEAN scale
20. Which of the following selection techniques has the highest predictive validity?

a) Personality assessment
PSYCHOMETRICIAN SET B

b) References
c) Structured interview
d) Unstructured interview

21.'Individuals are unique and complex. Their personalities are always changing and can never be
reduced to a measurement.' Which approach to personality does this best describe?

a) Nomothetic approach
b) Ideographic approach
c) Trait theories of personality
d) Type theories of personality

22.Which of the following is a factor which may cause bias in an interview?


a) Predictive validity
b) Halo/horns effect
c) Hawthorne effect
d) Nomothetic approach

23.Persona is
a. like a mask used by actors in a play.
b. only what others think about us.
c. a person's mechanistic behavior.
d. predictable and automatic.
24.For Freud, the basis of anxiety and neurosis is
a. parenting.
b. marriage.
c. sex.
d. hypnotic.
25.According to Jung, all of the following are major systems of the psyche except
a. personal unconscious.
b. ego.
c. collective unconscious.
d. libido.
26.Adler's primary research method was the
a. IQ test.
b. case study.
c. Style of Life Inventory.
d. word-association test.
27.For Horney, children develop feelings of security when
a. a parent stays home when a child is young.
b. parents show comfort, warmth, and affection.
c. no trauma is present in childhood.
d. a child becomes independent from parents at an early age.
28.At each stage of psychosocial development,
a. the mother is most important in resolving the crisis for the child.
PSYCHOMETRICIAN SET B

b. adaptive and maladaptive ways of coping are incorporated into the ego identity.
c. we must confront sexual and aggressive needs.
d. the maladaptive way of coping must be suppressed.
29.Allport believed that present and future behavior can best be explained in terms of
a. past behavior.
b. psychosexual conflicts.
c. functional autonomy.
d. instinctual drives.
30.The L-data technique uses
a. ratings of specific behaviors in real-life settings.
b. questionnaires.
c. observations of laboratory behavior.
d. objective tests.
31.The key idea in Maslow's hierarchy of needs is that
a. lower needs have to be satisfied before higher needs become influential.
b. individual needs are learned, not inherited.
c. safety needs are the most important of all the hierarchy of needs.
d. higher needs must be satisfied first.
32.Rogers's research on delinquent children showed that
a. family environment was the best predictor of later behavior.
b. genetic factors were more influential than environmental factors.
c. self-insight was the best predictor of later behavior.
d. it was impossible to predict later behavior.
33.To anticipate events correctly, constructs must be
a. general enough to encompass various events.
b. dichotomous or bipolar.
c. focused on the similarities among events.
d. based on exact replications of past events.
34.In behavior modification therapy, unconscious motivating forces are
a. extinguished.
b. brought to the surface.
c. ignored.
d. eliminated by negative reinforcement.
35.People who are low in ________ and ________ are much more likely to imitate a model's behavior
than are people high in these two areas.
a. IQ/grades
b. self-confidence/self-esteem
c. happiness/assertiveness
d. ego/superego
36.Zuckerman used the method of ________ to identify sensation seeking.
a. test-retest
b. item analysis
c. statistical reduction
d. factor analysis
37.According to studies of consistency in personality, ________ are consistent over the life span.
a. genetic influences
PSYCHOMETRICIAN SET B

b. habits
c. traits
d. environmental influences
38.Sigmund Freud viewed personality as
a) a concept unworthy for psychologists to study.
b) unconscious and hidden.
c) entirely dependent on temporary circumstances.
d) independent from the influence of childhood experiences.

39.A set of abstract concepts used to explain a group of facts or events is called
a) an empirical statement.
b) an operational definition.
c) a theory.
d) personality
40.Historically, formal discussion of theories of personality have been dominated by a(n)
a) Western focus.
b) feminist focus.
c) Afrocentric focus.
d) Asian focus.
41.Of the following questions, which most accurately reflects the study of personality?
a) How do humans differ from animals?
b) Who am I?
c) How have humans changed over the course of history?
d) When are behavior patterns determined?
42.Why are theories of personality constructed?
a) To help us understand human nature
b) To zero in on one general explanation
c) To sum up research
d) Because theyre necessitated by nature
43.John Watson recommended that psychologists should emphasize the study of
a) overt behaviors.
b) mental processes.
c) introspection.
d) covert behaviors
44.John Watsons extreme behaviorist views were adopted and taken even further by
a) Gordon Allport.
b) Carl Rogers.
c) Sigmund Freud
d) B.F. Skinner

45.Freuds theory of personality was developed on the basis of


a) rigorous scientific testing of human behavior in laboratory settings.
b) experimentation on animals, the results of which were generalized to humans.
c) his experience living in Europe, Asia, and Africa.
d) his own clinical observations
PSYCHOMETRICIAN SET B

46.The study of personality stems from two different approaches. What are these?
a) academic psychology and clinical practice
b) meditation and self-assessment
c) laboratory research on animal behavior and case studies on group behavior
d) social psychology and learning theory
47.What do personality theorists seek to accomplish in their role of scientists?
a) They deal with the study of ultimate reality.
b) They apply their findings to foster better living.
c) They explore what it means to be a person.
d) They test hypotheses to explain human behavior.
48.Of the following philosophical assumptions, which one concerns a persons ability to control his own
behavior and understand the motives behind it?
a) Freedom versus determinism
b) Heredity versus environment
c) Uniqueness versus universality
d) Optimism versus pessimism
49.The basic philosophical issue of optimism versus pessimism that personality theorists consider
centers on whether
a) people can change their personalities over time.
b) human nature is fundamentally good or bad.
c) hope is an essential feature of psychotherapy.
d) personality theories can adequately explain or predict human behavior.
50.Philosophical statements are ultimately tested by
a) rigorous experimentation.
b) perceptual observation.
c) empirical means.
d) their compellingness
51.Subjective data differ from objective data because
a) subjective data are not based on empirical data.
b) subjective data are more difficult to validate consensually.
c) psychologists are not concerned with subjective data.
d) subjective data refer to things outside the self.

52.Which of the following approaches to psychotherapy did Hans Eysenck advocate the most?
a) behaviorist
b) psychoanalytic
c) humanistic
d) cognitive
53.. The social cognitive perspective of personality theory_____________________
A. Emphasizes learning and conscious cognitive processes, including the importance of beliefs
about the self, goal setting, and self-regulation.
B. Emphasizes the importance of unconcious processes and the influence of early childhood
experience.
C. Emphasizes the description and measurement of specific personality differences among
individuals.
PSYCHOMETRICIAN SET B

D. Represents an optimistic look at human nature, emphasizing the self and the fulfillment of a
person's unique potential.
54. The trait perspective of personality theory __________________
A. Emphasizes the description and measurement of specific personality differences among
individuals.
B. Represents an optimistic look at human nature, emphasizing the self and the fulfillment of a
person's unique potential.
C. Emphasizes learning and conscious cognitive processes, including the importance of beliefs
about the self, goal setting, and self-regulation.
D. Emphasizes the importance of unconcious processes and the influence of early childhood
experience.
55. The humanistic perspective of personality theory __________________
A. Emphasizes the importance of unconcious processes and the influence of early childhood
experience.
B. Emphasizes the description and measurement of specific personality differences among
individuals.
C. Represents an optimistic look at human nature, emphasizing the self and the fulfillment of a
person's unique potential.
D. Emphasizes learning and conscious cognitive processes, including the importance of beliefs
about the self, goal setting, and self-regulation.
56. Thinking or behaving in a way that is the extreme opposite of unacceptable urges or impulses.
A. Projection
B. Denial
C. Reaction formation
D. Regression

57. A psychoanalytic technique in which the patient spontaneoulsy reports all thoughts, feelings, and
mental images as they come to mind.
A. Hypnosis
B. Free association
C. Introspection
D. Catharsis

58. A form of displacement in which sexual urges are rechanneled into productive, nonsexual activities.
A. Sublimation
B. Denial
C. Undoing
D. Rationalization

59. Retreating to a behavior pattern characteristic of an earlier stage of development.


A. Denial
B. Regression
C. Repression
D. Undoing
PSYCHOMETRICIAN SET B

60. In Freud's theory,, a child's unconscious sexual desire for the opposite-sex parent, usually
accompanied by hostile feelings toward the same-sex parent.
A. Identification
B. Oedipus complex
C. Ego defense mechanism
D. Penis envy

61. Children in Piagets concrete operational stage of cognitive development


a. are just beginning to develop their language abilities.
b. can use hypothetical reasoning and abstract reasoning quite well.
c. can think logically, but only about real objects.
d. are learning to understand the basic manipulation of objects.

62. The psychoanalytic persepctive of personality theory____________


A. Represents an optimistic look at human nature, emphasizing the self and the fulfillment of a
person's unique potential.
B. Emphasizes learning and conscious cognitive processes, including the importance of beliefs
about the self, goal setting, and self-regulation.
C. Emphasizes the importance of unconcious processes and the influence of early childhood
experience.
D. Emphasizes the description and measurement of specific personality differences among
individuals.

63. In Freud's dynamic theory of personality, the ______________ level of awareness contains
information that you're not currently aware of but can easily bring to concious awareness.
A. Conscious
B. Unconscious
C. Preconscious
D. Subconscious.

64. One major criticism of the _________________ and _______________ perspectives on personality
theory, is the lack of scientific testability and empircal evidence.
A. Trait, humanistic
B. Humanistic, psychoanalytic
C. Humanistic, social-cognitive
D. Social-cognitive, trait

65. Children in the _____________ stage will outwardly show a strong desire to associate exclusively
with same-sex peers.
A. Genital
B. Anal
C. Oral
D. Latency

66. Psychologist who believed that the most fundamental human motive is striving for superiority.
A. Carl Rogers
PSYCHOMETRICIAN SET B

B. Carl Jung
C. Karen Horney
D. Alfred Adler

67. Psychologist who stressed the importance of cultural and social factors in personality development.
A. Alfred Adler
B. Karen Horney
C. Carl Jung
D. Sigmund Freud

68. In Freud's theory, the _______________ level of awareness represents the thoughts, feelings,
wishes and drives that are operating below the level of concious awareness.
A. Unconscious
B. Conscious
C. Subconscious
D. Preconscious

69. Penis envy is a term used to describe_________________


A. A child's unconcious sexual desire for the opposite-sex parent, usually accompanied by hostile
feelings.
B. A man's desire to have a penis similar in size to those of African's and their descendents.
C. The sense of deprivation and loss a little girl exepriences when she discovers that boys have a
penis, and she does not.
D none of the above

70. Psychologists who believed that the deepest part of the individual psyche is the collective
unconcious, whihc is shared by all people and reflects humanity's collective evolutionary history.
A. Karen Horney
B. Sigmund Freud
C. Carl Jung
D. Alfred Adler

71. Theory of personality which emphasizes the importance of observational learning, conscious
cognitive processes, social experiences, self-efficacy beliefs, and reciprocal determinism.
A. Social cognitive
B. Humanistic
C. Psychoanalytic
D. Trait

72. Carl Rogers believed that the most basic human motive is the ___________________, which is the
innate drive to maintain and enhance the human organism.
A. Self-concept
B. Actualizing tendency
C. Conditional positive regard
D. Unconditional positive regard
PSYCHOMETRICIAN SET B

73. The ________________ is the set of perceptions and beliefs that you have about yourself, including
your nature, your personal qualitites and your typical behaviour.
A. Actualizing tendency
B. Conditional positive regard
C. Self-concept
D. Unconditional positive regard

74. The _____________ perspective stresses concious thought processes, self-regulation, and the
importance of situational influences.
A. Social cognitive
B. Humanistic
C. Trait
D. Psychoanalytic

75. In Roger's theory, the sense that you will be valued and loved even if you don't conform to the
standards and expectations of others is called:
A. Conditional positive regard
B. Unconditional positive regard
C. Actualizing tendency
D. Self-concept

76. Albert Bandura was a proponent of which perspective of personality?


A. Psychoanalytic
B. Behaviorist
C. Humanistic
D. Social cognitive

77. The beliefs that people have about their ability to meet the demands of a specific situation; feelings
of self-confidence or self-doubt.
A. Reciprocal determinism
B. Self-efficacy
C. Self-concept
D. Ego defense mechanism

78. A theory of personality that focuses on identifying, describing, and measuring individual differences
in behavioral predispositions.
A. Psychoanalytic
B. Humanistic
C. Trait
D. Social cognitive

79. One criticism, or weakness, of __________ theories is that they don't really explain human
personality. Intead, they simply label general predispositions to behave in a certain way.
A. Humanistic
B. Psyhcoanalytic
PSYCHOMETRICIAN SET B

C. Trait
D. Social-cognitive

80. People who rate high on the personality trait of __________________ tend to be upbeat, optimistic,
and sociable.
A. Introversion
B. Extraversion
C. Neuroticism
D. Psychotcism

81. A type of psychological test in which a person's responses to standardized questions are compared
to established norms.
A. Projective test
B. Inkblot test
C. Thematic Apperception Test
D. Self-report inventory

82. A relatively stable, enduring predisposition to consistently behave in a certain way.


A. Trait
B. Surface trait
C. Self-concept
D. Actualizing tendency

83. The most fundamental dimensions of personality; the broad, basic traits that are hypothesized to be
universal and relatively few in number.
A. Behaviors
B. Surface traits
C. Traits
D. Source traits

84. Psycologists who advocated the trait apporach to personality. His research led to the development
of the Sixteen Personality Factor Questionaire, one of the most widely used psychological tests for
assessing personality.
A. Hans Eysenk
B. Albert Bandura
C. Raymond Cattell
D. Carl Rogers

85. Personality characteristics or attributes that can easily be inferred from observable behavior.
A. Traits
B. Source traits
C. Surface traits
D. temperament traits

86. Proponenents of trait theory include


A. Karen Horney,. Albert Bandura
PSYCHOMETRICIAN SET B

B. Raymond Cattell, Hans Eysenck


C. Alfred Adler, Raymond Cattel
D. Hans Eysenck, Albert Bandura

87. A model proposed by Albert Bandura that explains human functioning and personality as caused by
the interaction of behavioral, cognitive, and environmental factors.
A. Reciprocal determinism
B. Self-efficacy
C. Trait theory
D. Sublimation

88. People who rate high on ____________ tend to be anxious, worried, and socially insecure.
A. Neuroticim
B. Introversion
C. Extraversion
D. Psychoticism
89.The final resoluation of the Oedipus complex occurs in adolescence during the ________ stage,
during which incestuous urges start to resurface, and the personb directs sexual urges toward socially
acceptable substitutes.
A. Latency
B. Genital
C. Phallic
D. Oral
90.In Erikson's theory of the eight stages of human development, at what stage would people most
likely encounter the psychological crisis of initiative vs. guilt?
A. Preschool
B. Elementary
C. Adolescence
D. Young adult
91. A sons hostility toward his father may masquerade itself in the form of excessive affection. This
mechanism, called reaction formation, gets activated because

A. the son does not want to hurt his own father due to social expectations;
B. the sons hostile feelings for his father are merely temporary;
C. in an undisguised form, the hostility would create too much anxiety for the son;
D. the sons hostility results from his inability to transform his feelings to socially desirable and
prized behaviors.

92.Rogers holds that psychological imbalance results from a disparity between


A. Past motives and future aspirations.
B. Active strivings and past fears
C. Conception of self and the real self
D. Approach and avoidance tendencies
93. According to the existential view, anxiety is seen as a:
A. Result of repressed sexuality
PSYCHOMETRICIAN SET B

B. Part of the human condition


C. Neurotic symptom that needs to be cured
D. Result of faulty learning

94. The philosophical assumptions underlying the existentail aproach include the notion(s) that:
A. People are thrust into a absurd world and their goal is put meaning onto it.
B. People must create their own meangings through their choices
C. Freedom is merely an illusion
D. Human destiny is mainly determined by external forces and social coniditioning

95. According to existentialists, all of the following issues are involved in our search for meaning except:
A. Discarding old values
B. Meaninglessness
C. Creating our own value system
D. Exploring unfinished business

96.Living authentically implies


A. Following the norms of our social group
B. Being true to our own evaluation of what constitutes a meaningful existence
C. Accepting responsibility for the fact that we create our lives by the choices that we make
D. Both (b) and (c)
97.Philosophically, the existentialist would agree that:
a. People redefine themselves by their choicse
b. A person can go beyond early conditioning
c. Making choices can create anxiety
d. All of the above
98.In social cognitive theory
a. people function as contributors to their own motivation, behavior, and development within a
network of reciprocally interacting influences
b. the balance and counterbalance of internal and external forces operate to influence both the
emotional and cognitive processes as the individual attempts to adapt to her or his environment
c. the order of importance of factors is indicated in the label: social first, cognition second
d. human functioning is emphasized because "cognition" precludes the consideration of animal
functioning
99.Situation that exists when a person is both attracted to and repelled by the same goal
ex: taking the job may bring in a significantly greater income but it is not a job you'd be passionate about
doing
a. Approach-approach conflict
b. Approach-avoidance conflict
c. Avoidance-avoidance conflict
d. Double Approach Avoidance Conflict
100.Mischel's cognitive-active personality system conceptualizes humans as
a. being motivated mostly by personal dispositions.
b. being goal-directed and active, not passively reactive.
c. primarily motivated by past experiences with rewards.
d. shaped largely by an interaction of variable personality traits and the situation.
PSYCHOMETRICIAN SET B

GENERAL INSTRUCTIONS:

11. This test booklet contains 100 test questions.


12. Read INSTRUCTIONS TO EXAMINEES printed on your answer sheet.
13. Shade only one (1) box for each question on your answer sheets.
14. Two or more boxes shaded will invalidate your answer.
15. AVOID ERASURES.

INSTRUCTIONS:

5. Detach one (1) answer sheet from the bottom of your Examinee ID/Answer Sheet Set.
6. Write the subject title INDUSTRIAL PSYCHOLOGY on the box provided.

Shade Set Box A on your answer sheet if your test booklet is Set A; Set B if your test booklet is Set B.

1. The following are the reasons in conducting training in a certain organization except:

a. Having new employees or newly hired applicants.


b. When organizational change comes.
c. Having newly promoted employees (rank and file to supervisor)
d. Having new organizational policies and regulations
e. None of the above

2. The ultimate purpose of the training is:

a. Enhance the employee performance


b. To help the employee to achieve the needed skill and knowledge for their job
c. Increase organizational profits.
d. All of the above
e. None of the above

3. The following are reasons why conducting needs analysis except:

a. To determine the types of training


b. The extent to which training is practical means achieving an organizational goals
c. To determine the KSAO as the basis of the training
d. All of the above
e. None of the above
PSYCHOMETRICIAN SET B

4. A properly conducted organizational analysis will focus on:

a. The goal of organization


b. The extent of training will achieving the goals
c. The ability to conduct training
d. A and B
e. All of the above

5. The organizations ability to conduct training will consider the following factors except:

a. Time
b. Money
c. Physical space
d. Manpower/ trainer/s
e. None of the above

6. Readiness of the employee for a training should be determined in:

a. Task analysis
b. Person analysis
c. Organizational analysis
d. B and C
e. All of the above

7. For a training to be effective, the following should be considered except:

a. Attendance of the employee


b. Performance in training
c. Application of training
d. Feedback of the training
e. None of the above

8. Employees are motivated to apply what they have learned from training if:

a. Employees are encourage and reward them to do so


b. The extent to which they are given the opportunity to apply the skills learned from training
c. Trainers will monitor them
d. A and B
e. B and C

9. The following are some strategies to motivate employees to attend training except:

a. Provide food
b. Reduce stress associated with attending
c. Allow feedback
PSYCHOMETRICIAN SET B

d. Increase employee buy-in


e. None of the above

10. Training goals and objectives should concretely state the following except:

a. The level at which they are expected to do it


b. What learners expected to achieve
c. The conditions under which they are expected to do it
d. The learners are expected to do
e. None of the above

11. The first step in developing the training program is:

a. To determine types of training


b. Establish the goals of the training
c. Establish the objectives of the training
d. A and B
e. None of the above

12. This is the common financial incentive method

a. Vertical skill plan


b. Skill based pay
c. Depth skill plan
d. Basic skill plan
e. Horizontal skill plan

13. The following are types of incentive can be used to motivate learning except:

a. Job security
b. Promotion
c. Fun
d. Self- efficacy
e. Self-improvement

14. Below are the factors in motivating employees to perform well in training except:

a. Interest
b. Providing incentives
c. Provide negative feedback
d. Provide positive feedback
e. None of the above

15. To determine the individual training needs for each employee the following are the methods use
except:
PSYCHOMETRICIAN SET B

a. Surveys
b. Observation
c. Skill test
d. Knowledge test
e. Critical incidents

16. The main disadvantage/s of surveys are/is:

a. Honesty of the employees


b. time consuming
c. affordability of suggested training
d. A and C
e. A and B

17. The main advantage/s of interview is/are:

a. Feeling and attitudes revealed


b. Data are often easy to quantify and analyze
c. Face to face interaction
d. All of the above
e. None of the above

18. Who developed the method Critical Incidents?

a. John Flagart
b. John Fermin
c. John Flanagan
d. John Lewin
e. None of the above

19. The following are the problems in using performance appraisal in needs analysis except:

a. Leniency errors
b. Strictness error
c. Either high and low on a dimension
d. Construction of performance appraisal
e. None of the above

20. Determining which employees need training and which areas will fall under:

a. Task analysis
b. Person analysis
c. Organizational analysis
d. Needs analysis
e. Analysis of variance
PSYCHOMETRICIAN SET B

21. The main purpose of task analysis is :

a. Determine the KSAO


b. Use the job analysis
c. Use the task inventories
d. Determine the task performed
e. None of the above

22. A topic on the training can be made interesting by:

a. Using humors
b. Maximizing audience participation
c. Having well known speaker
d. B and C
e. A and B

23. The most common job analysis method used for task analysis are the following except:

a. Interviews
b. Surveys
c. Observation
d. Task inventories
e. none of the above
24. Which factor does not contribute to the success of the performance appraisal review?
A. Keeping the results of the evaluation confidential from the employee
B. Scheduling the review to eliminate or minimize interruptions
C. Letting the employee discuss her feelings and thoughts
D. Mutually setting goals for improvements in future performance.

25. . In organizations not subject to employment-at-will, which is not a legal reason to terminate an
employee?
A. violating a company rule
B. inability to perform
C. beyond probationary period
D. part of a force reduction.

26. What is an example of a 360-degree feedback?


A. Behavior observed by supervisor
B. Behavior observed by supervisor, employee and customers
C. Behavior observed by supervisor, subordinates, peers, customer and employee
D. Behavior observed by supervisor and subordinates

27. What is a type of performance dimension that organizes the appraisal on the basis of goal to be
accomplished by the employee?
A. Goal-Focused Performance Dimensions
PSYCHOMETRICIAN SET B

B. Contextual Performance
C. Task-Focused Performance Dimensions
D. Competency-Focused Performance Dimensions

28. What is a type of objective measure?


A. Attendance Record
B. Personal Preference
C. Popularity
D. Attitude

29. What is a type of performance dimension concentrates on such employee attributes as dependability,
honesty, and courtesy?
A. Task-focused Performance Dimension
B. Goal Focused Performance Dimension
C. Competency-Focused Performance Dimension
D. Trait-Focused Performance Dimension

30. What is the easiest and most common employee comparison method?
A. Paired comparison
B. Forced distribution
C. Rank order
D. Individual evaluation

31. What method involves comparing each possible pair of employees and choosing which one of each
pair is the better employee?
A. Paired comparison
B. Forced distribution
C. Rank order
D. Individual evaluation

32. What type of error in evaluating employee performance involves the distribution of ratings on a rating
scale?
A. Central tendency error
B. Halo error
C. Proximity error
D. Contrast Error

33. What do you call that event when raters, who likes the employees being rated, may be more lenient
and be less accurate in their ratings than would raters who neither like nor dislike their employees?
A. Halo error
B. Bias
C. Observation
D. Leniency effect

34. What is a type of cognitive-processing of observed behavior?


A. Emotional State
B. Recency Effect
PSYCHOMETRICIAN SET B

C. Infrequent Observation
D. Sampling

35. Which of the following does not reflect a properly designed performance appraisal system?
A. Performance appraisal system is based on a job analysis
B. Performance appraisal system involves multiple behavioral measures of performance
C. Performance appraisal system is unstructured and informal
D. Performance appraisal system provides the opportunity for an employee to appeal

36. To obtain a measure of quality, there must be a standard against which to compare an employees
work. Which of the following is not a measure of quality?
A. A dressmakers work quality would be judged by how it compares with a model shirt
B. A teachers quality of instruction is judged by how the students are attracted to the teacher.
C. A secretarys work quality would be judged by the number of typographical errors
D. A cooks quality is judged by how her food resembles a standard as measured by size,
temperature, and ingredient amounts
37. In performance evaluation, which of the following is not considered an objective measure?
A. Attitude
B. Quantity of work
C. Quality of work
D. Attendance
38. A typing test was administered to 50 data-entry clerks at a particular organization. There was a
strong correlation between their scores on the typing test and their performance as data-entry clerks.
This type of correlational analysis refers most specifically to establishing:
a. construct validity.
b. criterion validity.
c. content validity.
d. concurrent validity.
39. If a small police department uses a cognitive ability test because a meta-analysis indicated cognitive
ability is the best predictor of performance in the police academy, it is:
a. going to see a reduction in quality.
b. predictive validty
c. using the Taylor-Russell method.
d. using validity generalization.
40. The Taylor-Russell tables provide an estimate of the ________ if an organization uses a test.
a. amount of money saved
b. liability
c. probability of future success for the employee
d. percentage of successful employees
41. A test predicts performance for one group of applicants (e.g., men), but does not predict
performance for another group of applicants (e.g., women). This exemplifies:
a. utility.
PSYCHOMETRICIAN SET B

b. differential validity.
c. known-group validity.
d. single-group validity.
42. A test predicts performance for two different groups of applicants (e.g., men and women); however,
the test predicts the work performance significantly better for men than it does for women. This
exemplifies:
a. single-group validity.
b. utility.
c. differential validity.
d. known-group validity.
43. The statement "It's not fair to pay a poor employee the same amount of money as an excellent
employee" exemplifies the need for accurate performance appraisals in:
a. making promotion decisions.
b. training needs assessment.
c. employee training and feedback.
d. determining salary increases.
44.________ ratings have been successful in predicting future success of promoted employees but/and
are ________ in organizations.
a. Self-appraisal / often used
b. Self-appraisal / seldom used
c. Peer / seldom used
d. Customer and subordinate / seldom used
45. According to most research, subordinate feedback results in:
a. higher supervisor turnover.
b. increased supervisor performance.
c. subordinates being fired.
d. bad feelings among employees.
46. Whereas trait-focused instruments concentrate on who or what an employee ________, task-
focused instruments focus on what an employee ________.
a. believes / feels
b. does / is
c. feels / believes
d. is / does
47. Which of the following employee comparison methods involves comparing each possible pair of
employees and choosing which one of each pair is the better employee?
a. Forced distribution
b. Behavioral anchored rating scale
c. Rank order
d. Paired comparison
48. The most common type of rating scale is the ________scale.
PSYCHOMETRICIAN SET B

a. behavioral observation
b. graphic rating
c. behavioral anchored rating
d. forced choice rating
49. Amy works at a bank that is trying to get customers to apply for vehicle loans. As hard as Amy tries,
she is unsuccessful because the bank's loan rates are not very competitive. When evaluating Amy's
performance, her supervisor must consider the loan rates as a:
a. performance dimension.
b. source of contamination.
c. modifier.
d. halo factor.
50. Documenting employee performance:
a. increases employee satisfaction.
b. decreases rating accuracy.
c. increases recency errors.
d. reduces legal liability.
51. The ________ affects the quality of ratings provided by raters.
a. amount of stress in the rater's environment
b. feelings the rater has toward the ratee
c. All of these are factors.
d. race of the rater and the ratee

52. A performance appraisal instrument would be most useful in terminating an employee due to:
a. a layoff.
b. employment-at-will.
c. an inability to perform.
d. a violation of company rules.
53. The performance evaluation is the most common way to determine raises or bonuses. Which
statement about subjective measure is correct?
a. they are direct, quantitative, measures of performance
b. they are limited by situational factors and not always useful for complex jobs
c. they rely on the judgment of the rater
d. they do not measure important aspects of motivation and cooperation
54. There are different types of Subjective rating techniques. Which statement out of the following ones
refers correctly to BARS?
a. BARS are job behaviors associated with very good or very bad performance
b. BARS is used for comparing some of the employees to other employees
c. BARS is rated on several dimensions of the job performance
d. BARS present rater with statements grouped by social desirability and successful job
performances.
PSYCHOMETRICIAN SET B

55. Which of the following SUBJECTIVE RATING TECHNIQUES has the similar advantages and
disadvantages of the BARS?
a. FCC
b. BOS
c. PCS
d. KSAP
56. The are several Rater Biases. From your knowledge about these biases, find the correct definition of
the Halo Effect.
a. tendency to assign average ratings to all rates
b. tendency to give all rates positive ratings
c. tendency to judge all aspects of behavior on a single attribute
d. tendency to rate by comparisons to others.
57. Why is the rank-order method used in the performance appraisal of employees?
A. To encourage competition
B. To reduce leniency
C. To increase reliability
D. To have a valid comparison
58. The following appraisal dimensions are organized based on goals to be accomplished by a police
officer. Which is not a goal-focused performance dimension?
A. Knowledge of the law
B. Prevent crimes from occurring
C. Arrest lawbreakers
D. Minimize citizen complaints
59. Only few organizations allow an employee to evaluate her own behavior and performance. The
reason is that:
A. self-appraisals tend to suffer from leniency.
B. self-appraisals are highly related with actual performance.
C. self-appraisals are used for salary increase or promotions.
D. self-appraisals are very subjective.
60. Subordinate feedback is an important component of 360-degree feedback in performance
evaluation. Subordinates will be discouraged to provide feedback if:
A. Supervisors appear open to employee comments
B. There will be retaliation for making honest ratings
C. Ratings are made anonymously
D. Ratings are used for developmental purposes
61. Assuming you are a supervisor. You have decided to include peer rating in performance evaluation.
Why?
A. Peers see the actual behavior of employees
B. Peers report directly to supervisors
C. Peers are good sources of information
PSYCHOMETRICIAN SET B

D. Peers are comfortable with fellow employees


62. Why is supervisor rating the most common type of performance appraisal even if supervisors do not
see everything that the employee does?
A. Supervisors are seniors and are therefore the appropriate persons to evaluate employees
performance
B. Supervisors see the end result even if they do not see every minute of an employees
behavior
C. Supervisor rating is the most simple and direct method of performance evaluation
D. Supervisors are in a position to evaluate
63. In order to obtain an accurate view of job performance, organizations use some form of multiple-
source feedback and 360-degree feedback. Based on this, who should provide feedback when evaluating
a bank tellers performance?
A. Branch managers
B. Customers
C. Peers
D. All of the above
64. What should you not consider in the performance appraisal process?
A. Use of subordinate rating when supervisor is needed to appraise.
B. Design an elaborate performance appraisal system if supervisors are highly overworked,
C. Develop a numerically complex system when which there is no money available for merit pay
D. Use of peer ratings in an environment in which employees are very cohesive
65. Amy used a method of controlling for order effects by giving half of a sample test A first, followed
by test B, and giving the other half of the sample test B first, followed by test A. this method is called?
a. Alternate-Forms Reliability
b. Counterbalancing
c. Split half
d. form stability
66. The most important use of performance evaluation data is to provide _____________ to the
employees and assess her strengths and weaknesses.
A. Information
B. Feedback
C. Analysis
D. All of the above
67. In making decision on which of those applicants are to be hired according to their test scores, one
needs to use________________ if there are 9 openings of that position and rank their scores from
highest score and going down until it fills the 9th slot. That 9 applicants will be considered hired

a. Unadjusted top-down
b. Rule of three
c. Passing score
d. Banding
PSYCHOMETRICIAN SET B

B d. Applicants choose their own references.

68.Which of the following is NOT true regarding the use of grade point average for employee selection?
a. All three are true.
b. GPA predicts best in the first years after graduation.
c. GPA has low adverse impact.
d. GPA predicts job performance.
69.Cognitive ability tests are excellent predictors of employee performance, but because they almost
always result in ________, they should be used with caution.
a. misdiagnosis
b. adverse impact
c. invasion of privacy issues
d. high costs
70.A selection method that requires data-entry clerk applicants to actually enter information into a
computer using a particular software package is MOST accurately referred to as:
a. a cognitive ability test.
b. job knowledge testing.
c. a work sample.
d. an assessment center.

71.Which of the following assessment center techniques allows the applicant to demonstrate such
attributes as creativity, decision making, and ability to work with others?
a. In-basket technique
b. Business games
c. Out-basket technique
d. Simulations

72.The 16PF is a personality test whose 16 dimensions were determined by a factor analysis. The 16PF is
an example of a(n) ________ test.
a. statistically-based
b. parapsychology-based
c. empirically-keyed based
d. theory-based

73.Which of the following psychological tests or selection methods is LEAST likely to predict the criterion
with which it has been paired?
a. Cognitive ability / performance ratings
b. Interest inventories / employee performance
c. Personality tests / sales
d. Biodata / tenure
PSYCHOMETRICIAN SET B

74. When personnel professionals are concerned with adverse impact or invasion of privacy associated
with a test, they are assessing the:
a. potential for legal problems.
b. reliability.
c. scoring methods.
d. cost.

75The human resource manager at Robson Machinery is looking for a method to hire employees that
will predict future performance and have little adverse impact. Which of the following would you most
recommend?
a. Graphology
b. Personality inventories
c. Integrity tests
d. Cognitive ability tests

76.. Wonderlic Personnel Test and Raven Progressive Matrices are example of:

a. cognitive ability test

c. personality test

d. interest test

e. job knowledge test

77.. Kristian Malaluan is applying for an HR position. On his resume he declared that he completed his
course at UP Diliman and obtained 1.5 GWA. He also indicated that he worked as call center agent
during his college year. Since Kristian lied on his education and work experience, he engaged on what
we called _________________.

A. Negligent hiring

b. Resume Fraud

C. Dishonesty

D. Resume Fake

E. None of the above

78. John, the recruiter, asked the applicant for an HR position on how to conduct job analysis. What type
of employee selection did John apply?

a. cognitive ability test


PSYCHOMETRICIAN SET B

b. job knowledge test

c. ability test

d. work samples

e. Personality test

79. What is the most important thing to consider when writing rejection letter?

a. dont beat around the bush

b. be honest

c. use positive scripting

d. thank the applicant

e. be as personable

80. An applicant for telephone operator will allow sitting before a switchboard to handle a distressed
caller who is describing an emergency situation. An applicant must properly answer the call, calm the
caller and obtain the necessary information in a little time as possible. This type of exercise that can be
found inside the assessment center is:

a. in basket technique

b. simulation

c. leaderless group discussion

d. work sample

e. business game

81. Thematic Apperception Test and Rorschach Inkblot Test are example of what type of test?

a. projective test

b. objective test

c. interest test

d. integrity test

e. intelligence test

82. The following are ethical guidelines that reference providers follow except:
PSYCHOMETRICIAN SET B

a. explicitly state your relationship

b. be honest in providing details

c. let the applicant see your reference

d. always seal the reference before sending to employer/HRD

83. Kim would like to measure the communication skill or a certain applicant both oral and written.
What type of employee selection test will Kim used?

a. Cognitive ability test

b. psychomotor test

c. job knowledge test

d. physical test

e. work sample

84. Mayi is looking for a test to the applicants for machinist and die maker. She needs to know how good
these applicants in discrimination colors and glare sensitivity. What type of test should Mayi used?

a. perceptual test

b. pychomotor test

c. physical test

d. job knowledge test

e. cognitive test

85. The type of employee selection test are most likely used in the retail area.

a. personality test

b. integrity test

c. interest test

d. cognitive test

e. intelligence test

86. An applicant for a job as an automotive mechanic asked to fix a torn fan belt. This type of employee
selection process is:
PSYCHOMETRICIAN SET B

a. work sample

b. job knowledge

c. cognitive test

d. perceptual test

e. psychomotor test

87. The following statements are true about interest inventory except:

a. SII is an example of interest test

b. Interest test is used in vocational counseling

c. Individual with similar interest to those of people in particular will more likely be
satisfied in that field

d. Employees whose interests are not congruent with those of the job are performed well.

88. Physical Ability tests have been criticized on the following major points except:

a. validity

b. job relatedness

c. passing scores

d. when ability must present

89. Carlo, an HR officer, would like to know the applicants life history. He asked the company
psychologist to conduct interview to know more about applicants childhood experiences. This type of
employee selection process Carlo applied is:

a. personality test

b. integrity test

c. psychological exam

d. drug testing

e. medical exam
PSYCHOMETRICIAN SET B

90. The _______________ is the process of determining the work activities and requirements, and
_____________ is written result.

a. job description/ job analysis

b. job evaluation/ job description

c. job analysis / job description

d. job analysis / job evaluation

91. The job analysis method uses written reports of good and bad employee:

a. ergonomic job analysis procedure

b. critical incidents technique

c. PAQ/ person analysis questionnaire

d. Occupational Information Network

92.. Employees referred by successful employees had___________ than did employees who had been
referred by unsuccessful employees

a. more motivation to work

b. longer tenure

c. more job satisfaction

d. performed well

93. Salary surveys are used to determine:

a. internal equity

b. external equity

c. merit

d. comparable worth

94. The best method(s) to use in analyzing a job appears to be:

a. the Position Analysis Questionnaire


PSYCHOMETRICIAN SET B

b. interviews, observations, and job participation

c. The Critical Incident Technique

d. dependent on how the information will be used

95. Creating a list of tasks that are thought to be involved with a job, and having job incumbents

rate the tasks on scales such as frequency of occurrence and importance, best defines

which job analysis method?

a. Task analysis

b. b. Critical incident technique

c. Job-element approach

d. Ammerman technique

96. Which of the following is the most common method of conducting a job analysis?

a. Observation

b. Task analysis

c. Interview

d . Job participation

97. Amalia is writing job descriptions and can't decide whether she should include "getting coffee for the
boss" as a task. Her decision is related to the issue of:

a. level of specificity b. formal vs. informal requirements

c. the Peter Principle d. the DOT Code

98. Informal changes that employees make in their jobs is called:

a. job crafting

b. spontaneous revision

c. position alterations

d. job reconstruction
PSYCHOMETRICIAN SET B

99. Which of the following sections in a thorough job description can be used in help wanted
advertisements, internal job posting, and company brochures?

a. Job title

b. Brief summary

c. Work activities

d. Work context

100. Psychologists involved in _______ study leadership, job satisfaction, and employee motivation.

a. personnel psychology

b. organizational psychology

c. training and development

d. human factors
PSYCHOMETRICIAN SET B

GENERAL INSTRUCTIONS:

This test booklet contains 100 test questions.

16. Read INSTRUCTIONS TO EXAMINEES printed on your answer sheet.


17. Shade only one (1) box for each question on your answer sheets.
18. Two or more boxes shaded will invalidate your answer.
19. AVOID ERASURES.

INSTRUCTIONS:

7. Detach one (1) answer sheet from the bottom of your Examinee ID/Answer Sheet Set.
8. Write the subject title PSYCHOLOGICAL ASSESSMENT on the box provided.

Shade Set Box A on your answer sheet if your test booklet is Set A; Set B if your test booklet is Set B.

1. If a mental ability is to be given to Filipino adults living in the highly urbanized city and asks a
question as a part of information subscale: Who is the 6th President of America? she said item is
can be best described as:
a. Culture free c. Internally consistent
b. Reliable d. Valid
Best answer is A. Reliability is consistency, Valid is accuracy. Internal consistency is item analysis
2. If the court issues a subpoena to a psychologist should:
a. Tell the court that he will release after he has secured consent from the client. ( Subpoena is a
summon from court to present document though it is mandated by the law you still have to ask
for informed consent)
b. Tell the court that he cannot do so as said data are confidential
c. He will obey the court with mental reservation
d. He will readily obey the court as part
3. Who demands for the following ethical standards like informed consent, confidentiality,
competence??
a. Examiner ( test user) c. Professional organization
b. Examinee( test taker) d. Researcher
4. Which is the concern of a test developer if he is trying to establish construct validity:
a. His object is determining the extent to which the test is related to specific criteria.
b. He is concerned with how well a test measures the construct that it claims to measure (
measuring what purports to measure)
c. He is focused on how well a test represents all aspects of the construct that if claims measure
d. None above
5. The area of a normal distribution from 0 to +3 encompasses approximately of the curved:
a. 99% b. 68% c. 95% d. 50%
PSYCHOMETRICIAN SET B

Right side above the mean of bell curve is 50% to the other side is 50%= 100%
6. Which of the following is content related evidence of validity?
a. Observed participants characteristics are consistent with responses on a scale( domain)
b. Scores obtained on two administrations of the instrument are consistent ( test retest or parallel
form)
c. Scores are corrected with scores obtained on another instrument (
d. Test items are at an appropriate reading level ( face validity)
7. If not accompanied by further information, a high raw scores is:
a. Reliable when correlated with another variable
b. Valid when correlated with another variable
c. Always better than a low score
d. Meaningless ( cant interpret or cant compare)
8. When transformed into the Wenchsler scale type of deviation IQs, a Z score of -1.00 would
become a Wenchsler IQ of :
a. 115 b. 85 c. 105 d. 95

Wenchsler IQ has the mean of 100 and standard deviation of 15.


9. The client who is trying project fake good or fake bad is an attempt which tries to manipulate
others. This process is called:
a. Discriminates evidence ( two opposite evidence)
b. Divergent thinking ( two opposite thinking)
c. Discriminant analysis ( two opposite analysis)
d. Impression management (

10. _____ constitutes the most widely used frame of reference for test score interpretation:
a. Criteria c. Content domain
b. Norms d. Work samples
11. The following statistical techniques may be employed for establishing internal consistency except?
a. KR20 c. Chronbachs alpha
b. ANOVA ( use for to know the difference of the mean of 3 d. Spearman brown
12. The following reliability has content sampling as a source of error, except:
a. Split half c. Equivalent, delayed
b. Test, re test d. Parallel, immediate
13. The best construct related evidence of validity comes from:
a. Validity coefficient that is near 0
b. A series of studies of the instrument
c. Careful review of the instrument by expert
d. Data based predictions that prove to be correct
14. Test items or question with three or more alternative response are:
a. Polytomous test item
b. Dichotomous
c. Standard error of measurement
d. Item response theory
15. If the shape of the distribution of scores obtained from a test is significantly skewed, it means that
the test is probably ____ for the takers:
a. Too hard c. Either too easy or too hard
PSYCHOMETRICIAN SET B

b. Too easy d. Just right


16. A trait a variable, such as reading ability, creativity or attentiveness:
a. Z score c. Measurement
b. Test d. Construct
17. Which of the following statement about measurement using continuous scales is true:
a. Always involves error
b. Is always ordinal in nature
c. Is always normally distributed
d. All of the above
18. Which of the following statements a accurately portrays the relationship of reliability to validity?
a. Score reliability is not related to inference validity
b. Scores must be reliable before inferences can be valid
c. Inferences must be valid before the scores can be reliable
d. The more valid the inference is, the higher reliability of the score must be
19. A measure of variability in data indicating the distance from the lowest to highest score in the set:
a. Range c. Semi interquartile
b. Mode d. Standard deviation
20. A test developer wants to correlate score obtained in a self-esteem inventory he has constructed
with his two kinds of respondents: students active in extra n curricular activities and students who
are not. The test developer is pro9bably establishing:
a. Validity c. Reliability
b. Norms d. All of the above
21. A pearson correlation coefficient describes the ___ and the ___ of a linear relationship between two
interval scale or ration scale variables:
a. Level; amount
b. Direction; strength
c. Similarity; importance
d. Variability; significance
22. It refers to the list of respondents in a particular area who agree to respond to questionnaire
regarding products, services, advertisements or other promotional efforts:
a. Consumer panel
b. Consumer
c. Motivation, research method
d. Consumer psychology
23. The doctor reviewed a measurement scale designed to measure extroversion and said that it
appeared to measure extroversion. The measure has ___ validity.
a. Internal c. Convergent
b. Face d. Predictive
24. Compared to psychological testing, psychological assessment is generally;
a. More structured c. More expensive
b. Simpler d. More objective
25. Validity is demonstrated when a measure is NOT related to variables with which it should not be
related.
a. Concurrent c. Discriminate
b. Convergent d. Differentiation
26. Content validity:
PSYCHOMETRICIAN SET B

a. Construct validity
b. Prima facie face validity
c. The emotional principles of a psychological test
d. A test with only one digit numbers
27. If its plural form, norms is a term used in psychometrics to refer to the test performance data of:
a. Distributing norms to members of target populations
b. Putting a carpenters personal signature on a work product
c. Test takers who constitute a control group in an experiment
d. People tested at a different time that another group of test takers
28. My measure allows me to successfully predict future behavioral outcomes. My measure has:
a. Face validity c. Coefficient of stability
b. Criterion validity d. Alternate reliability

29. One way to assess the stability of measuring, computing a Pearson:


a. Linear coefficient
b. Coefficient criterion
c. Coefficient of determination
d. Correlation coefficient
30. Which of the following statements about test anxiety and test performance is correct?
a. The relationship between test anxiety and test performance is direct
b. The relationship between test anxiety and test performance is indirect
c. The relationship between test anxiety and test performance is non linear
d. The relationship between test anxiety and test performance is non directional
31. Which of the following is a projective technique?
a. Army beta c. Bender test
b. NEO PI d. Block design
32. Equal to the square root of the averaged squared deviation about the mean, this statistics is called:
a. Average deviation
b. The range
c. The semi interquartile range
d. The standard deviation
33. Which of the following characteristics of the bell shaped curve tells that the lines do not touch the
baseline?
a. It bilaterally asymptotic
b. It is asymptotic
c. It is asymmetrically asymptotic
d. It is symmetrical
34. The MMPI gives a quantitative measurement of an individuals:
a. Emotional adjustment
b. Reality testing
c. Internal psychodynamic adjustment
d. IQ and mental health
35. The theoretical basis for the Edwards Personal Preference Schedule is :
a. The psychodynamic approach by Freud
b. The factor analytic approach by Cattell
c. The needs system proposed by Murray
PSYCHOMETRICIAN SET B

d. The socio cultural perspective proposed by Horney


36. The Minnesota Multiphasic Personality Inventory in a (an):
a. Personality test developed mainly to assess job applicants
b. Personality test used primarily to assess focus of control
c. Empirically derived and objective personality test
d. A projective technique
37. A test that was developed for a particular use with a particular population is said to be:
a. Sensitive c. Standardized
b. Reliable d. Valid
38. The personality test Mary is taking involves her describing random patterns of dots. What type of
test is she taking?
a. MMPI c. A projective test
b. MBTI d. An empirically derived test
39. He coined the term mental tests:
a. Lewis Terman c. James McKeen Cattell
b. Raymond Cattell d. Alfred Binet
40. It can be argued that all validity measures are measuring:
a. Predictive validity c. Criterion validity
b. Content validity d. Construct validity

41. In contrast to a power test, a speed test:


a. Tends to yield a score differences among test takers that are based on performance speed
b. Tends to yield spuriously inflated estimates of alternate forms of reliability
c. Can yield a split half reliability estimate based on only one administration a test
d. Has a time limit designed to be long enough to allow all test takers to attempt all items
42. A keys differences between the terms psychological testing and psychological assessment is that
psychological testing refers to process that:
a. Is more technician like that psychological assessment
b. Was the first described by Maloney and Ward in the mind 1970s
c. Is much broader in scope than psychological assessment
d. Involves more problem solving than psychological assessment
43. A five point Likert scale is most likely to commit:
a. Severity error c. Central tendency error
b. Random error d. Halo effect
44. When thirty out of forty test takers answered item no 7 correctly said statement is describing:
a. Item difficulty c. Item discrimination
b. Validity of the item d. Reliability of the item
45. A psychological report should:
a. Use jargon
b. Be at least ten pages long
c. Be read only by the client
d. Directly and adequately answer the referral question
46. The deviation IQ is based on the notion o0f:
a. A delinquency as a factor affecting the development of intelligence
b. Mental age versus chronological age
c. Verbal versus performance scores
PSYCHOMETRICIAN SET B

d. A z score
47. The area of the normal distribution for a T distribution encompasses approximately of the curve:
a. Ninety five percent
b. Fifty percent
c. Ninety nine percent
d. Sixty eight percent
48. An online IQ test that provides result based on the performance on a single task, such as block
design, is said to be a poor intelligence test even if it correlates well to school success because it is
said to have low:
a. Content validity
b. Reliable validity
c. Criterion related validity
d. Concurrent validity
49. A senior instructor at the Air Force Academy insists that his personnel test for officer candidate
school need on, consists of one question: Did you ever fly a model airplane that you built yourself? If
this one item test was actually used to select their candidates, we could assume that the test:
a. Based on informal empirical criteria keying
b. Based on formal factor analytic procedures
c. Invalid due to the informal nature
d. None of the above
50. Which of the following self report inventories include items for measuring mental ability?
a. 16 PF test c. MMPI 1
b. NEO PI R d. MMPI 2
51. Evidence of psychological testing can be traced back to:
a. Hammurabis code of civil law
b. Public service examinations in ancient China
c. Selection of candidates for the Roman senate
d. Caveman challenging each other to lift heavy stones
52. Which of the following is true about internal consistency reliability?
a. Its measurable using alternate forms
b. Its measurable via test retest format
c. Its measurable using then split half procedures
d. It measurable the consistency over time or situation
53. Empirical keying refers to:
a. Empirically validating test scores via research
b. Scoring a test based on the theory of what is being measures
c. Scoring a test based on its ability to discriminate between certain identifiable groups of people
d. Scoring a test using a scoring key made out of cardboard in which small holes reveal the correct
answers
54. What is psychological assessment primarily used for:
a. Report writing
b. Legal decision making
c. Answering referral question
d. Research

SITUATION: For problem 58 60


PSYCHOMETRICIAN SET B

Researchers have noted that a decline in cognitive functioning as people age (Bartus, 1990) However,
the results from other research suggest that the antioxidants in food such as berries may reduce and
even reverse age related declines (Joseph, et.al). To examine the phenomenon, suppose that the
researcher obtains a sample of n=16 adults who are between the ages of 65 and 75. The researcher used
a standardized test to measure cognitive a functioning for each individuals. The participants then begin a
month program to which they receive daily doses of blueberry supplements. At the end of the two
months period, the researcher again measures the cognitive performance for each participants.
55. What kind of experimental design was used?
a. Within subject design
b. Pre experimental
c. Between subject design
d. Quasi experimental
56. Which of the following threats of internal validity is more likely to be encountered by the
researcher:
a. History
b. Instrumentality
c. Regression toward the average
d. Both a and b
57. The Semantic Differential is basically a technique for the measurement of ________.
a. Honesty
b. Verbal fluency
c. Evaluative connotations
d. Perseverance
58. The difference in test performance between percentile scores of sixty and fifty five:
a. Cannot be readily equated to all other five point differences in percentiles
b. Is equivalent to that between percentile scores of sixty and seventy
c. Ids equivalent to that between percentile scores of fifteen and ten
d. Is twice that between percentile scores of fifteen and ten
59. Result for a client on an psychological test:
a. Should be interpreted in isolation
b. Should be interpreted by a computer
c. Should not be interpreted in isolation
d. Should not be interpreted by a computer
60. Projective test are not objective because:
a. Different scores are likely to produce the same test score from the same test performance
b. Scoring is heavily dependent on the judgment of the score
c. They are based on responses to ambiguous stimuli
d. They are scored in a simple, straightforward in a simple, straightforward manner
61. Mental status examinations what type of assessment?
a. Structured interview
b. Work sample
c. Portfolio assessment
d. Behavioral observation
62. An psychological test can become obsolete when:
a. Psychological theory develops to render the basis of the test obsolete
b. Society changes to render the content of items less appropriate
PSYCHOMETRICIAN SET B

c. Society changes to render the tests norms obsolete


d. All of the above
63. Objectives tests are objective because:
a. Different scores are likely to produce the same test score from the same test performance
b. Scoring is heavily dependent on the judgment of the scorer
c. They are scored in a simple, straightforward manner
d. They are based on response to ambiguous stimuli
64. Another way of taking about the reliability of a test for a particular purpose is to talk about it:
a. Utility c. Dependability
b. Validity d. Discriminability
65. Systematic error in attest exerts what kind of effect on test scores?
a. Consistent c. Unknowable
b. Random d. Inconsistent
66. Multiple choice test provide more than two options for each question to overcome the problem of:
a. Faking c. Defensiveness
b. Carelessness d. Guessing
67. Forensic assessment is considered different from therapeutic assessment because:
a. The purpose of assessment s different
b. The validity of the tests used is different
c. The tests used for assessment
d. The time required to conduct assessment is different
68. Which of the following areas are usually covered in a Mental Status Examination?
a. Appearance, orientation, affect through content and process, insight
b. Appearance, orientation, tendency to lie, though content and process, insight
c. CT scan, appearance, orientation, affect, insight
d. Orientation, affect, thought content and process, insight, CT scan
69. In forming a test can become obsolete when:
a. How the test is to be used
b. What we expect the average respond to be
c. How we ensure everyone in the sample is normal
d. How we ensure the sample is normally distributed
70. A false positive in personal selection is:
a. The rejection of an applicant who could have been successful
b. The use of a test with negative validity
c. The appointment of someone whose job performance turns out to be substandard
d. The incorrect scoring of a psychological test leading to someone being appointed who shouldnt
have been
71. A test of scholastic attitude is administered at the beginning of first semester and the academic
performance of the sample is examined at the end of the first year of university. (I.e two semester
later.) A failure to find a high to perfect correlation between test scores and academic
performance
a. indicates the test lacks concurrent validity
b. indicates the test lacks predictive validity
c. may indicate a lack of test validity but may also reflect intervening effects unrelated to
scholastic aptitude
d. is highly unlikely
PSYCHOMETRICIAN SET B

72. Which of the following procedures does not yield an estimate of the reliability of a test?
a. Finding the average of the correlation of each item with every other item.
b. Correlating each item with the total score on the test
c. Correlating the total of items in the half of the
d. Correlating the total of all even numbered items with the total of all odd numbered items.
73. Which of the following statements about norms is correct?
a. The standard deviation of scores on intelligence tests has remained constant over time
b. The raw score mean on intelligence tests has been decreasing over the years
c. The raw scores mean on intelligence tests has remained constant over the years
d. The raw scores mean on intelligence tests has been increasing over the years
74. Mata analysis is a term used to describe:
a. A tendency of scores to fan out from the mean
b. A graphic technique for representing regression
c. A variant of psychoanalysis developed by Can Jung
d. A method of combining information across studies
75. The personality test Maria is taking involves her describing random patterns of dots. What type of
test is she taking?
a. An empirically derived test
b. MMPI
c. MBTI
d. A projective test
76. When 35 out of 40 tests takers answered item no. 7 correctly said statement is describing:
a. Item difficulty c. Item discrimination
b. Validity of the item d. Reliability of the item
77. Test scores can be meaningfully compared when:
a. The test or test versions are the same
b. The test and test versions are different
c. When reference group are different, so long as test versions are the same
d. When derived scores are the same, even when test are different
78. Cultural sensitivity in test development is manifested by:
a. Panels of experts who review the test items for discriminatory content
b. Test purchaser qualification form that require statements of cultural background
c. Behavioral evaluation of test scores from different cultural background
d. Strict adherence to the guidelines to form in Tara off v Regents of University of California
79. If Pearsons correlation coefficient is zero, then two variables are not related:
a. True c. Neither true or false
b. False d. Either true or false
80. A researcher obtained the following regression equation from a sample of data: y = 5 + 2x. Using
this equation & an x value of 3, her prediction to the corresponding y value is:
a. 30 b . 11 c. 8 d. 6
81. With regard to the samples used to established within group norms, the single most important
requirement is that they should be:
a. Gathered locally by the institution or organization that will use them
b. Very large, numbering in thousands
c. Representative of the group for which they will be used
PSYCHOMETRICIAN SET B

d. Convenient to obtain in the process of standardization


82. Which of the following statements about test retest reliability is false?
a. The reliability coefficient of a measure should be at least 80% based the measure is accepted as
reliable
b. Sometimes test retest reliability is artificially high because people remember how they
answered the first time
c. Test retest reliability is the best form of reliability measurement
d. Test - retest reliability is very useful if the variable being measured is expected to stay constant
over time
83. Which of the following is NOT true of nonstandardized tests?
a. Nonstandardized tests do not have standardization samples
b. Nonstandardized tests do not have norms
c. Nonstandardized tests cannot be used for psychological assessment
d. Nonstandaridized tests can be psychometrically sound
84. Test construction, test administration, and test scoring and interpretation are?
a. Source of error variance
b. The sole responsibility of a test publisher
c. Facets according to the true score theory
d. Variables affected by inflation of range

85. Which of the following psychological tests make use of a theory of personality in its construction
and development?
a. Edwards Personal Preference Schedule
b. Minnesota Multiphase Personality Inventory
c. 16 Personality Factor Test
d. Guilford Zimmerman Temperament Survey
86. A psychologist developed an instrument will determine successful marriages. Using the said
instrument she found that 85% of those who scored a T score 70 remained marriages for about 20
years and above and 75% of those score below 30 got annulled of separated in the bed and board.
The psychologist concluded that the test has high:
a. Reliability c. Normative data
b. Validity d. Standardization
87. The following is a speed test, except:
a. SRA b. RPM c. CFIT d. PNIT
88. The ____classmen people according to Carl Jungs personality types:
a. Myers Briggs Type Indicator c. MMPI
b. Locus of Control Scale d. TAT
89. A persons accumulation of stored information, called _____intelligence, generally with age.
a. Fluid; decreases
b. Crystallized; increases
c. Fluid; increases
d. Crystallized; decreases
90. Which statement is true?
a. Validity sets the limit on tests reliability
b. The validity of a test limited by its reliability
PSYCHOMETRICIAN SET B

c. The validity of a test is limited only by the tests inter rater reliability
d. Validity sets a limit only on a tests inter rater reliability
91. Which of the following is not a Level C test?
a. MMPI 2 c. Bender Test
b. Wechsler Scales d. Rorschach inkblot Test
92. In a psychological testing and measurement, who is a user a test/psychological test?
a. Examiner c. Examinee
b. Test constructor d. Test developer
93. Which of the following procedures does not yield an estimate of the reliability of a test?
a. Correlating the total of all even numbered items with the total odd numbered items.
b. Correlating the total of items in the first half of the test with the of items in the second half of
the test
c. Finding the average of the correlation of each item with every other item
d. Correlating each items with the total score on the test
94. The term social desirability when used with respect to construction of a personality test refers to the
fact that?
a. People differ in their tendency to create a favorable impression of themselves when answering
items.
b. People differ in how strongly they are drawn to the company of others
c. People differ in how attractive they find social activities
d. People differ in terms of their tendency to agree rather that disagree with personality
statements
95. Transforming scores on psychological tests is done primarily to:
a. Protect the privacy of the test taker.
b. Add interpretation of the scores.
c. Make the scores more manageable.
d. Make the scores available for research.
96. Test construction:
a. Is a linear process with one stage following the other without variation.
b. Is a relatively inexpensive process.
c. Follows the sequence of steps but these steps may need to be retraced from time to time.
d. Can be done quite quickly.
97. Which of the following are affected by cultural factors
a. verbal communication and nonverbal communication
b. nonverbal communication
c. verbal communication
d. equal protection under the law
98. Test developers do which of the following to ensure that a test developed for national use is indeed
suitable for national use?
a) they have a panel of experts review the items
b) they try out the items and eliminate those that are biased
c) they have a panel of experts review the items AND they try out the items and eliminate
those that are biased
d) they employ a large number of examiners
The right to have test findings held confidential by psychologists can be found in:
PSYCHOMETRICIAN SET B

99.t est-takers have the right:


a) :all are correct
b) to know the results of the test they took
c) to know how their data will be used
d. to know why they are being tested
100. which of the following is a satisfactory way to minimize language effects of a written, standardized
test in English when administering the test to members of a Japanese-speaking group who speak English
as a second language?
a. prior to the formal test administration, have the examiner conduct a brief tutorial in English,
specifically answering questions about the meanings of words in the test items
B. none are correct
c. have a professional translator read the test to the group, simultaneously translating the items
word-for-word
d. have a member of the group who is fluent in English and Japanese read the test to the group,
simultaneously translating the items word-for-word

101. The Flynn effect refers to the observation that:


a. The raw score mean on intelligence test has remained constant over the years.
b. The raw score mean intelligence test has been increasing over a years.
c. The raw score mean intelligence test has been decreasing over the year
d. The standard deviation of scores on intelligence test has remained constants over time.

102. Incremental validity is psychological assessment refers to _________.


a. The extent to which new information increases the accuracy of a classification or prediction
b. How much the test contributed to the understanding of the client
c. How predictive is the test in forecasting future tendencies and behaviors
d. How accurate is the test in assessing psychological characteristics
103. Which is NOT part of the six stages of test construction?
a. Defining the test
b. Selecting a proper method
c. Constructing the items
d. Developing the test
104. In this stage of test construction, the scopes and purpose must be stated before the test
developers can proceed.
a. Revising the test
b. Selecting a scaling method
c. Defining the test
d. Publishing the test.
105. Which of the following coefficient represents the strongest degree of correlation between
variables?
a. 80 b. +1.20 c. 0 d. +.60
106. The form class interval is best associated with?
a. A socioeconomic status of a sample of test taker
b. A frequency distribution of test taker scores
c. A grouped frequency distribution of test taker scores
PSYCHOMETRICIAN SET B

d. Measure of central tendency


107. Which is a criterion referenced test?
a. The final condition in the Your Face Sounds Familiar
b. An examination for entry into a faculty union
c. A teacher made bid term examination scored on a curve
d. All of the above
108. This validity is demonstrated when a measure is NOT related to variables with which it should not
be related?
a. Convergent c. Concurrent
b. Differentiation d. Discriminant
109. An assessment center is
a. A place where assessments are conducted
b. A place where assessments information is collated
c. An index of central tendency for a large number assessment
d. Comprised of many different activities
110. An examinee who sacrifice speed for accuracy emphasizing?
a. Quality over quantity
b. Quantity over quality
c. Reliability over validity
d. Relevance over determination
111. Which of the following statements is correct?
a. A phi coefficient is used to establish the relationship between the score on polytomous scale
and a dichotomous
b. A phi coefficient is used when establish the relationship between there those who answered
true in every item and the total score in the 10 item quiz
c. A phi coefficient is used when establishing relationship between the response in the
dichotomous scale among the male and female respondents
d. A phi coefficient is used when establishing relationship between the response in the
polytomous scale of participants from urban and rural locality
112. It is considered as the breakthrough in the creation of modern tests.
a. The development of the Woodworths Personal data Sheet
b. The development of the Simon Binet Intelligence Test
c. The use in China of the tests for selection of public officials
d. The use of the tests in Greece to further educational system
113. Convergent validity is intended to explain what relationship is?
a. Premises from conclusion
b. The degree to which an operation is similar to other operations
c. Scores on a test can be correlated with scores on any other test
d. High correlations between random test scores.
114. It refers to the average of all test scores in a distribution.
a. Mean b. Mode c. Median d. Percentile
115. Coefficient alpha is also known as:
a. Cronbacks Alpha c. Kronbachs Alpha
b. Cronbachs Alpha d. Chronbachs Alpha
116. The most frequently used coefficient of correlation is:
a. Standard Deviation c. ANOVA
PSYCHOMETRICIAN SET B

b. Pearsons R d. Chi square


117. An attribute, trait or characteristics from overt behavior is called:
a. Construct b. Concept c. State d. Standard
118. It is the machine that measures the brain activity while asleep.
a. Electromyography
b. Echoecephalograph
c. Electroencephalograph
d. Sphygmomanometer
119. What is the machine that measures electrical activity of the muscles?
a. Electromyography
b. Echoencephalograph
c. Electroencephalograph
d. Doppler
120. These are the abilities which consist of nonverbal and culture free.
a. Crystallized Intelligence
b. Three Stratum Theory of Cognitive Abilities
c. Fluid Intelligence
d. Emotional Intelligence
121. A theory of intelligence that relies on how information IS PROCESSED.
a. Information Processing Style
b. Cross Battery Assessment
c. Two factor Theory
d. Correlation Matrix
122. The collection of items to be further evaluated for the final version of the test is
a. Item Pool c. Item Analysis
b. Item Bank d. Item Sampling
123. The middle Score in the distribution is
a. Mean b. Median c. Mode d. Range
124. It is a symmetrical distribution of scores that when graphed looks like bee shaped
a. Norm b. Normal curve c. Mean d. Mode
125. It refers to two or more forms or version of the same tests.
a. Parallel forms c. Pattern analysis
b. Homogeneity d. Heterogeneity
126. Memory for how to do things or perform certain functions called __________
a. Declarative Memory c. Procedural Memory
b. Working Memory d. Long Time Memory
127. It is method or procedure for evaluation or scoring:
a. Protocol c. Assessment
b. Profiling d. Norming
128. It reflects a simple tally or basic score calculated in a test performance:
a. Rating Scale c. Range
b. Raw Score d. Performance assessment
129. The extent to which the test is self consistent or stable is called ___________
a. Reliability c. Validity
b. Scoring d. Interpretation
130. The science of psychological measurement is:
PSYCHOMETRICIAN SET B

a. Biospychological c. Biospychosocial
b. Clinical d. Psychometrics
131. Analysis that provides information on two variables or group is called___
a. Bivariate Analysis
b. Multibariate Analysis
c. Exploratory Factor Analysis
d. Measurement of Central Tendency
132. A is to A as to O is a./b B c.>d. O. this is an example of hypothetical is called:
a. Aptitude test c. Achievement test
b. Personality test d. Interest test
133. CAPA is an acronym that stands for:
a. Computer Assisted Psychological Adaptation
b. Computer Assisted Psychological assessment
c. Career assisted Potential Aptitude
d. Checklist Adaptive Personality Assessment
134. Which does not belong to the group?
a. Criterion Referenced Testing Assessment
b. Domain Referenced Testing and Assessment
c. Content Referenced Testing and Assessment
d. Norm Referenced Testing and Assessment
135. Which of the following is NOT included in measuring the ratio IQ?
a. Mental Age c. Chronological Age
b. 100 d. Individual Performance
136. A laboratory study was designed to research a phobia of snakes in the wild. This is an example is:
a. Severity error
b. Central tendency error
c. Discriminant analysis
d. Leniency error
137. He was the one who developed the two factor of intelligence in 1904.
a. Howard Gardner c. Jean Piaget
b. Charles Spearman d. Alfred Binet
138. Which does not belong to the group?
a. Tailored testing c. Sequential Testing
b. Branched Testing d. Objective Personality
139. Which does not belong to the group?
a. Item Response Theory
b. Latent Trait Theory
c. Latent Trait Model
d. Item Characteristic Curve

140. These are specific and distinguished features or characteristics of an individual.


a. Character b. Personality c. Attitude d. Trait
141. It refers to the standard measure of performance against which a test or a score is evaluated.
a. Construct c. Cognitive Test
b. Criterion d. Cultural Relativity
PSYCHOMETRICIAN SET B

142. It is a family of statistical techniques which is used to shed light on the relationship between certain
variables.
a. Method of Predictive Yield
b. Mapping Method
c. Discriminant Analysis
d. Job analysis
143. It refers to a tool that is used to pinpoint students difficulty or area of deficit usually for remedial
purposes or intervention.
a. Achievement Test c. Diagnostic Test
b. Aptitude Test d. IQ Test
144. What does DSM 5 stand for:
a. Diagnostic and Statistic Manual of Mental Disorders, Fifth Edition
b. Diagnostic and Statistic Manual of Mental Disorders, Sixth Rendition
c. Diagnostic and Statistic Manual of Mental Disorders, Fifth Rendition
d. Diagnostic and Statistic Manual of Mental Disorders, Sixth Edition
145. It is a group of tests hand picked by the assessor relevant to some purpose.
a. Flexible Battery
b. Performance Assessment
c. Fixed Battery
d. Clinical Assessment
146. It focuses more on informal learning or life experiences.
a. Aptitude Test c. Achievement Test
b. Perceptual Test d. Power Test
147. It is an error in measurement when the test taker does not possess a particular behavior or trait
when in fact the test taker does.
a. False Positive c. False Negative
b. True Positive d. True negative
148. The practice of norming on the basis of race or ethnic background is called___
a. Age Norms c. Grade Norms
b. Race Norming d. Local Norms
149. In test construction, it is the process of setting rules for assigning numbers in measurement.
a. Validity b. Reliability c. Range d. Scaling
150. Which of the following is NOT included in the precautions for safeguarding test records?
a. arranging for the storage of records for eternity
b. locking the file cabinet
c. determining when records will be found outdated
d. determining when records will be used only for

e) they employ a large number of examiners


PSYCHOMETRICIAN SET B
PSYCHOMETRICIAN SET B
PSYCHOMETRICIAN SET B
PSYCHOMETRICIAN SET B
PSYCHOMETRICIAN SET B
PSYCHOMETRICIAN SET B
PSYCHOMETRICIAN SET B

You might also like